Sunteți pe pagina 1din 49
1. Introducere Johann Peter Gustay Tejeune Dirichlet (13 februari 1805 - 8 mai 1859) ‘fost un mare matematician german. Familia sa provine din Rieelette, un sat din apropistea oraulul Lege din Belgla. De sci provine ¢ mumee st Lejeune Dirichlet (Le Jeune de Richelet, care i franca inseam Tira dé Fichelte). Ss niseat in localitatea Dire, ote aat tangs Kl, in German pe atamel alas in Imperial Fences condos de Napoleon, Tata sit font Fancionse poeta. CCursurile gimnacile Ye wrmeag Is Bonn, ia clea Koln, ‘A urmat exrsoril universitare ia Franta la Pais, incepdnd ox an 1822, shsoivind Facullates de Stinje de la Collépe de Franer- Ait a intra in contact ‘ov mari matematicini ai timpull eum arf Legendre, Laplace sau Poul. event in Germania eta Iuerat cu Gauss cu Jacobi. Obgine un doctor ‘norte la ecomandaresIui Alexander von Bumbo, c tea ck masta lniba Jatin, conditeobligatorie pe atunetpentra obtinerea unl doctorat. Prod la Universitatea din Brest, atiot Wroklaw, Polo, dae gi ln Uni- wrvitatea din Berlin timp de 27 dean neinterapt, ‘A amt pretene deo vith eu Jacobi, aoeta din rm vind o puternich lntuents action sa Prantee ce mal taut levi ais amintin pe Leopold Kronecker (1823+ 1801) x Bernbasd Reunann (1826-186). [1858s pling dsipoluuisiu Kronecker de prea multele sole itor «ste supa. Jn 1850, dup moartea Iai Gaus, succedeacestuia la catecen de In Universdates din Gattingen, anal din marile centre matemaice ale hm, liberinduse asl de multe soir eeora use sus ‘Se poate dedicn mai mult timp cereetri dar, din pate, fa timp une\ expuneri la Montroatx, a sufst un atne da im Bol sale grave |e adagh ‘moarten ote. Mone in § ma 1859 la Gttingen (Cu Dirichlet incepe epoca de aur & matematil la Bats, Inteodvore ‘S.aocupat de mute capital ale matematiel, dar ese ral importante conte ‘ni lew avut tn teria numerelr, anal i teria potential Prima sa coneibujie importants ln matematics const in demonstraree ‘Mari tooreme al Fermat” pentru cial = 6 ‘A demonstrat eX daca gb sunt mumere inte prime inte ele, tunci srl (n+ Jeon confine oininitate de namere prime. Aces rzultat ete cunoseut sub rumete de Teorema fui Dirichlet. Demeonstrati pe care Dict m dao ‘ces Voteme in aml 1837 a ps baoee tore analies anmercor Prin introdnoerea sero atagte fotilonarlmetee, age mumitle seit Dinchlt ada ocontibuje important la elaborates insiramentelor de aca pentru teori modern In teria potential se ocup cu problema existent! file anmonice cae ai poerl nude. Tot ela dat gh condiia Diet prvind convergentsseilor trigonometric, Principal Dirichlet este exnoset ub denuslreade Principe des tein Pranta (stick Principid sertarelor, Pigeonhole principle in englezs (Principia ‘upilor de porate), Schulfachyinaip in germank saga mult Prine cutie sat Principal sertarlor. In cuda wna enunt simply, are o muljime de aplinti in diveee cepiole ‘lo matematic, Putem inal probleme de In cele mn simple pin le p= ‘lene netriviale care pot ft ezalvatereeurgind la acest Poucipin, de multe oi In stuati surpesestoare. Ditichlet La wtliet in studil eorpul mumereie alebrce. ‘Act princpin se bavened peo observaie simpli st amume cf, dacs trebuie 6 pune niste ble i» ext, sumkru tile Bind rai mare decit al etl, om obliga sk punem fa cel palin una dint eat maint deo Blk (0 cutie contin col putin douk bil). Inteade, dack ence cutie ar contine ea mul o bi, am avea‘un mum de hile ol mut egal eu numdraleutior, ceca ‘ne fal. Agadar puter enunta uniter princi: Principal tui Driclet: Daca asem m enti care repartsim m 41 bile atunci wa exita ed putin o cue care a4 confind el pofinP ble Cu variant Daca avern n cation cre repurtisim cel pufin n+ 1 bile tamed vs existe cel pujin 0 cutie crea coin el pun 2 be. Important do ficut oleate 2 8 poste procian cate ete cut care are cel putin dou bie, m ve poate precizn care unt acelen gm puter yi eite dea sin abstract Putem demonsra urmitonres generaliare Principia ui Dirichlet genenaizat: Dacd inn cuit pmem p+ ble atuned © cutie va contne cal putin p +1 tie, Demonstrate: Prespinenoyfcte dite oa neti ws ol mp, ten ‘stl vr malt rn il. Cotsen wt os He adevirata, Tote de crak Patan egy ne con dt: ear leper ec pre cl induce nse _ Stl mca plo pene gs chen pans en ci cculoare si mirime); ° ck ah a 8 pean, pen f gw ek dk dnt ele ut niscute in acteagi ai a sAptiimanii; en “ ak oe 18 pate et8 pain do este ace ot « Motinin ein aw ote bgatrve et peas seced Pitta principal In rezolvren problemeler cu ajuoril Principle Dvclet const ina sabi care sun "eat" i eave mun "ile Wallace ponte conduce la situa eudat, dlr fcevente tn matemalc, in eave atin ‘kopre cova ct exist du fiw! putea identi, ‘Spre oxempln putem da onunjur allecoative ale sees princi. Brung 1's Fie A 0 muljime nevid, ar Ay, Aa, y Ay 0 partir Inteodvoare (aia @ pentru i) Oana ier ADA, acd avem 0 colstc de n +1 elemente ale mulfimi A, atunci exsta i € (1.2,..n} asl incie doud dinte cele m1 elomente sf se ofle in tuljince n wos ca cle m multe le partie! represin cutie ar cle n 1 element considerate, din maltinen 4, represint isle. ‘Enung 2: Fie A gi B douk malin finite de cardinal m respectivn. Atune: 1) Dac m > n atune mu exist ofuneteinjetiv deni pe A ca valor in b. 2) Dac m < m abun exists o Fanti njctv dar mu gi Bietivi, Se pot demonstra uritoaree: Fle Aji B dons itn finite avind m, respect m eemente.Atunck: 1) Numaul function f= A> Beste n™, 1b) Dac v =m, atanc mira fneyilor jective fs A + este ml: 1) Daek tm , atuoe! nama funeilae surjective f A> Bete: = Chin FER — 2)" — CH= 3) 44 ACE nung 9 « Pie A si B dou mami de candinalen gi respectio m, stil thoi n > m, Atuncs exit ae B ast ined If-Ma)| 22 nung 4 Dect raprtim m1 obiote fmm ext atunc exist o cute ‘are confine niet un oiet (0 ali form "taro" a Principia cutie ests ie queda mamere postive Entre, Dac abot tm mtl birt sunt ditibuite San cui abun fe prima cute confine minim a abiecte, fe dowa minim gy oy fe a n-a mi dy Demonstrate: Preupunvan mene devia, deci ctia cu muri contne maxim q = 1 obit, t= Fx, Akane in ode neti vor Wel nll (qh — 1) Introdocere (0 = 1) 4 + (p= 1) = 44 $494 + gy —M Bnd mal mle en 1 dec cole considerate. Am git astfelo conteadicho ‘Forma simpli a Principia util se ofine din acca fieknd gy = 92 = = a2. Atanch Gitta tq-ntl=2n—nglandl Porma "tare" a Principia cutie esta fost mal ales pente eal ay = du = K Ia sonst exe principil este: * Dac n( = 1)-+1 obiect se distribaie in m cut. Atul oe! putin o cute confine eel putin Kobiete ‘Su forma echivalenth= ‘Fem mimere natural stil init totes pet ‘Agel cel putin unal diate acesten esto mai mare sau egal cu PPatem emunfaeitevateoreme ale Principiul cute vlan infin * Dack repariai un nimi finit de abc iatrun mmr infil de etl, unctun mami infin de enti wor rime goale * Dacd repartiim un muri infit de obiceteinte-un mum Hit de et, uyor oeatie wa contin un nue fit de ble, * Dac repattai un numi inf de obec intr-un nami infnt de eutlh ‘wy pte spe nimle dackineS ra am Hut © "irarhaare” a frelon fn colo co wrnoass von presenta aplicai ale Principinl cutie in dow evans Algebrs ¢ Coometse. La inceputal Baral eaptol worn aes 0 ar ‘i probleme restate apat vom propune spre rezokare an set de probleme ae ‘me rpunsue sau slug le von prezenta La ina 2. Principiul cutiei in Algebra 2.1 Probleme rezolvate A, into cast sunt 8 de clevi, Si se arate eh exist elev nfceui in scoot is Sohutle: Dac ar fi niscuji maxim 2 elev in Gecare din cele 12 juni ale lui atune i clas & masz 24 de elev. Contra! Obcrsoie: Nu este obligator sa fe nlccug eel putin 4 elev into fund, corse exist un contraexemplu: Sunt ete $ niacin feare din primele 1 nial anual nictunal in ces de-a dousspreaezea Ins. 49 aflenumiral minim de elev a une elase penta af sigur sont ora cl pln 3 in acs a, olutie: Daek se § mai putin de 24 elev intro clasé am putea sbi xopae- Viiv maxisn ete 2 in feeare hunk a anu. Dac sunt 26 acest Incr na ma ote psi. Agsdae mimeo! minim este 25 de elev Olvera: Pentru af sigori exist Into class ost pun 14 Lele In aceasi hd ar trebu sh em minim 12 +1 elev noe cash. A Dialog: ‘A dot cumple 6 pixuri de aceny eon. Nu cred eae posbil deoarece mai am doar 17 plxur gh unt de 4 cur dferite. Principia enti fo Algor cine diferente, Fie seston by < by < 68 Atuncl, in eax ch nc nu © stax Tlie eonlazi,cferentele by ~b, By ~ by by ~ by 36 aft im wulimen C. Dar by = (yb) + (ta) 1, Arta oricare «mums real pst exst mumerele ne poative m ti att nee Iza) ce Solutio: od an ntreg at ine 5, ae nest {ar)~{on) Dar {or} = or~(ax] (br) = [es notin m =a sire lar = ober ceigs 12, Plo 11 numere rele mai sui co gi mal mil ex 32. Sie arate ch ests ‘tel Inte le ear pot nga Inter unui tinnght, Solute: Se partitonesahintrvalal (1;82) in intervale (1522 SA Conform Princip cuit tei dintre exe 11 mumee "bilele, s gsr int ‘ntl lute ele cnc intervie, "euile”. Feo, Osc aceste mumere situate in Interval 2 2°), Ok <4, Bridet, deoarece a-+b> 4-2" = 21> ¢ | ameticl, ny lo elite tefunghia Generate: Fi 2n-+1 name vale mai mati det 13 mai mici dee 2 Si yo ante exit tel diatt ele care pot @ lunge Itarbr unui engi Hea Vou imple intrvaul (152) tm interval, "euile, iar cele 2n-+ 1 numere hill”, prin eepartiacen fn ot, rn gk tre dintre ee acl in- terval, pin ure stisficind conluia 18, Si se determine numira mas de mimere rene dif, wenule, ear at propretaten ck din oricare 7 dinkre aestesputem ale donk care wu suman Solutio: Arian cf 12 ¢ munkrul etna Ps marl repost a a a Vor cme ‘Ax (4us;209;..;-ag). Arkt cl orice submulfimea ei, care contine 7 eleme fre dont care ax summa 0. “ ms Porm gase "catit” {a}, /= 17 inte cai, prin wrmae su sua nals. ack luke cel pata 18 mumere real, conform Principia cute, gapte din- leo acetea au acest seen, sdack Ie alegem pe acest sigur suma oricaror duit ‘ste ie postiv dak cele 7 sunt postive, fe negativs,dac& cle 7 sunt negative ous diate cele 7 bile we af atm Gene Sw etc sin de mae le i, venue area popitaten 8 in ericare n+] ince eoren pte alge dou care au sum oui. u Putem formula un ena ase: ‘Sse determine mamral maxim de mumerereole poitive si diferite care a propeitaten oi, din oriare 2n 1 dine acestes, putea alege dou care ai Prodan eg 1 1. Oricaro aumis rational se serio sub frm 2ocimalt Bit san periodic Solugir Fie numérul rational $, ag Integ, 62 2; (dock b= 1 ntunel a ‘ot facia zociald tat). fmpaztim pe al 8 obtinem: $a242 pate Ai cle rina acini at: fain poo 10! tmp a Ohyinen: si De neces hat - wo Analg caleulim urmtonrelozecimale (nn plerdem din vedere ek mu sunt enplra fo, e pt f numer). (Const eX zacimalee sunt etule ucceviv obtinae prin mpetees Ia b sturilor de la etapa peeeadent multe 10. Dacia um moment dat ratul oO, stune toate resturile ulterior sunt 210 ‘dae operata we opreste; facia este zcimald fat. Dac mu, operatia 9 continu pink cid constatimn ef scimalele ee repet ooarece: “Zectalele mu pou fim numir mai mare decit cele 8 restusi case pot bint pin impistires un ame ince In b ‘Are he antral reratal: ‘Un numa este zonal dack numa dak srieren sa subform soins ote fit sau puoi. 16, Cie 1, 2 8, ou 950 Smpart tn tel multi. SK se arate ob exit us ‘ate are prods! cementlor eel pin 72 4 Solu: Produsul clr 9 cfte ete 362 880 care este mai mare decit 70% Dect na din submit are prodasul elmentslor el putin L. Dae prods tis pont exact 71 dee va 8 ni 72 20, Un grup de 67 levi este tstat cu un set de 6 Stree munerotate de Jn 6 Penteu un ricpuns coect la intebarea cu mumieuln (n= 16) 9° ‘ punete. In cial uni rispuns gest se peed n puncte 8) Cove este cen mai mick diferent pontv8 posi fatre dowd pun Aiterte? ') Dovodii ek xl putin 4 participangi au obsnut aeagt puncta, «) Dove ek ca pain dot paseipangi au dat rspunets!Klentic la Intrebirile. (0.N. 2002) ‘Solutio: a) Orce elev primeste wn puneta deforma: Halt $6 ato neo acrongl pavitate ou 1-424 +6 = Bt, dee! impat. Agadar diferent nin nn poate 1. Pentru a aria cb pote f 2, dim un exemplu concret Hlovul Anu a resohat probleme 1,2, 8, 4,5 gt rezlvat pe gases are net eval 1 ou 8 rezlvas probleme 1, 2,3, 4,6 sf a reatvat problema 8; are muctajo 0; 1) Pometajal nim este-21 iar cel mas este +2, namai umes immpare ec in total 22 posible punctaje fete. Find 67 eles, conform Prinipuld ‘ule, pte let vor ave acelai punts, ©) Cun la fear din ele 6 intebi aver dou vastante de pons (ore Incoret) vom ava 2° = 52 varnnte de repuneurtdiferite (ets diferente 2 Probleme rezolvate epunsutidieite si punetaje dori). Decl vor exata dot peticipanti eu ‘lag mod de hapa 17. Fie gin mere naturale, p> 21m > Iga p. Pio maljime den numero loop {015025--)¢q)- Demonsrei ef exist douk submltin distinete A st 1 ale acest malta afl init Heels i Go oii prin p Solutle: Avem din jpotee Asada, conforin Principia cute, vor exsta dou submaltia, fe acs ‘Asi B, sector produse ale elementelor daw acalas est prin imptire la Atanein diferena lor edb pein p 10 2.2 Probleme propuse 1, Sivae arate eXoricare af 12 numere ditinete de dowt ce, exist dou a ‘itor diferent ete wn namie deforma a, 2. Sion arate ci, orieato arf pte mumere péteate perfecte, exist douk a ‘irr diferent este malin de 10, 3. La un dinen particpk 15 persoane. SK se arate ck exist 2 persoane care a ‘uelgh mune de eunoging peite eat participant. (Se considers cf dc A eaoagte pe B atunet gi B exogts pe 4 Lau turnen de baschet paticipf > 2 echipa Sind ci oricare peeche lo chipesinronté mas @ dati ose arate ck exist doa ecipe cae jones ‘sealag! numa de paride. (Variant m exert entero! + Ioteun grup sunt 17 prletens. Numele sf prenamele lor tacep numa ct Ia A, 18,6, D. Demonstrate exist el putin doi prietnt ale ekror nite ‘onl. emonsraj ek orcur am aloge$ aumere naturale pe care le dic ta teres 4n,printre mimereleobjite gsm Intotdesnna el potin douk eare fy acoagt ulti ees. 1. inten sheng sea 100 bie avind una din euorl cicolorul 48) Demonstrai ci din bite sconce lant mplare exist el pan dul de aecast ‘iloare +) Cae este ural minim de ile co trebule extras ponte af sigur et avem fel putin 5 de aoeag lone? 1% Si ae arate ok, dace leg 55 de mumere Integ istnese din interval [1,200 exists pereche de mumere a eitor diferent eato 10,0 alth peecho a ‘int dilereng ete 12, dar e pol 8 nn existe o erect a etl iereng 8 fet 1. Demonsteali print orcare m ume ite din interval [,2n ~ 2] ‘st ot ing care au sma impari. 10, Tour cls sunt 90 lov, tind ea un tot Stefan a Seu 12 gree iat 4 ‘ila au feu feeare mai putine greg docdt esse arate cl exit elev eo coal mame de greg 11. Ste aate ck orcam am alege 51 de mamere in multines (123.100) ‘int printe ele dud eare sin prime inte ele 12. Si se arte i orcare ar fun numkr impor a exists un num natusal 6 Wel inet 2° — 1 98 fe lip de a 18. Fe ayaxt.jaip mumere naturale diferite, Si se arate eX exist prinio (wn oomsr sa eteva mumere conscative care an sna dvi ew 10 Mle A o oubmultine de 50 elements multi (1.2.., 100} afl inet Wewe don clemence ale submalti an sume diferits de 100. Arita el A oline un pirat perfect 18. ‘Sise arate ct intr-un grup de gase persoane exis rei care sunt pietone Ince cle orizare doud sau sunt stdin intre ee orice dou, 16, So dau gave puncte, ericare trl neeniniare, Uncle dnt aceste pte 96 ve douk te donk. SK se arate ed exist tei puncte care sunt fe unite latte We oncare dous, fe oricare dou dinte le nu sunt unite 7. ‘So dau save puncte oricare tre necliniare, Uni oricare douk puncte gi (olor Gcaredintre eqgmentele obtinute ea una din eure nb sa rou. $k ovate exist un triunghi care ae latuile colorate ew 6 acongteulore, 1%, in 800 de cui se afi mere. Se tio Xin Hecaze cute sea cel mule 240, ‘Sse demonsteze ed exist eal potin 3 enti ce contin aclu aut de 19, fntr-un sac s afi 100 bile avind una din eulorileticoloeub 1) Demonstrag edn bile soase fa ntampare exist. el putin do lar, 1) Cae este suairu minim de bile cebu extrage pentru af suri even «ol putin 5 de seen culoase? de ncengh 20. ”Si.se arate of orieuim am alego cine namere tates, exist dow dint evn, enze a Sumas diferent vibe eu 7 1» 21. Se comidrtroce mumere naturale de cite douk cir (sctise in bara ‘rc. Sits demonstreze ci din acste rimnere se pot foros dou grupe evade syle satel tet sunele manera din Sacre gape eae (OAL tora p 2. Sides mmr rtrd td ie oi nba Sts dott nase sees ptf de pep le Sa int nat ste namin rae et 23 Fe § 0 miline formats din gote mumere intel cups ere 3h 2, Dovel ck ma elenestlor din el pain do eed ae ta 9 ee tae Greate 24. Flo 01, dav © petmutare & numerelor 12,9. $8 se demonstree ch podual (1 ~ (42 ~ 2).fag ~ 9) ese par. 25. Genertsare exer’ anter. Feo ate 0 permite a gumerelor 12, unde n mumir ipa. S& se lemons pred) " mrss (41 Mea ~2)foy =n) ase par 26. intro cate sont 10 erefoane dle culare roy, 8 de culoave alban, 8 de ‘aloare verde a 4 de culoare galbenk Din cut te extrag n rcioane, ‘S8 ¢ ‘etrmine nomral winim de ereionneearo treble extrac, stl Inedt ob fe no uni pun de # ereioane de acseasieasare. 2S arate prints ake m4 same ia mie (125.2 pneu ‘unul pe celia. cana i: Dak ge on ns Seen or een, Peempi: mt Hint 25.0520 on ai Se pte cr ete vie pia doa or egt ‘esto o patere natural lui 2. m - velar Sho eta hrc slog mene ar ic nce 1 ei doa dr a ca "ores Chen" ir ib (Conceal "Force. Compan 6 2. Generazare a probleme’ rezouate 6. ‘Seditun numir natural k > 1 Sire determine el mat mie numike natural n x proprietates urmitoare: Oricum am alegen mumereintreg, existe pain "ht printreoceston caze a summa su diferent dive sim 2k + 20. Fie A o mmuhime formatk din 19 mumere inte, distincte dou ete oni, eareapartn maltimi 1, 4,7, 10, 100}. S& se arate exist dol ine Anetta A'acieorsumé este 104 (Putnam) 30. Sie arate printre m+ 1 numere naturale nema difrte mat inc su tiple det 2 ae glee tre mumere cu proprctates ck nul dnt ele este summa febrile dou 41. Avem 2n-+1 bile mmerotate en numere de a ln 2n +1. Care cel ma nave nae de be pe care puter slege afl int nil unl dinicemnmere ‘hu fe ogal eu sume altor dou. 2. Si s0 arate ok ereum am alege n+ 2 numer din rnin {22,3}, Prints ee exis dou’ eu diferent stent in interval (20) Fle 2-1 anmese rele mal mari dect 1 smal mii docit 2". SS se arate ‘ens prnte le eet mumere care pot fungi Iturtr unui tiunghi Martini sunt og, veal san alba. Ei pot avea ince 2 gi § minis de Jn la 20 de ante, Care este mumiral mini de marten necesar pent a f tiguri ck po f slectagi 11 marten dente? 5, Bie S o mulfime de numer intreg prime ex 10, ma micl dost 301, Im © 2}. Gist cel mai mic k astfl incat orice submultime A.C 8 de k slemente confine dens numere inte distnete, astfelincit um i divide pe lia, sk arte exist dou peintze ele a chror sui ete mai mare dec 8. S17. 0 persoant cum 45 eS trun interval de 30 aie. $tind fin Hrcare zi eumpirat eel putin o carte, we arate ck exist un amnmit mumir dele consecutive In are cumpeat exact 14 A Priseipileutil Algo 38, Pie 50 de puncte tn plan, orcare tei ncolniare, Acsste puncte sunt trate ear eu ua dnte ede pata cule avute le dispose, Demonstra ‘eh exist o eaoat, gf eal putin 130 de tfunghiur salen, eu vAfulle printre ppuncelecolorate eu eu aceasta eon (Baleaniada pentru Junie, 2007) 0. Lao peieecre portcip 10 perce (604 soe). Acegia da mana wait cu li (ov neapirat fecare cu fears) La nal unul date bicbai intr pe ‘ill eu ete perane on dat mda. Sind ef ma eu extat douk spun entice, sh se alex cite personne a dat mana stia sa. (Onear dow parse dat mina col molto dat inte de smu existh porsoani eaves dea mina, ‘3 nsf ne persoand nu get on snl et sti). 40, Demonstrate penta orice num Spar a, exist un murat natural inl Paste not vide 2 — 1, 41, Demonstrate, pentru ore mami natural n, exist mami format fg ile 1 9h 2 eae se divide eu 2, io numerela Inte aa, 02, 35 oy QM % 2. SH se demonstrene ch, ht el m ner notte by, ba, By mm, ate net musa oc Be divid x Hip arate ovate arf 10 mumece naturale consecutive, exit printre nul are oto prin eu elelate nou. 44. floriem po shad 44 de monet naturae nemule dstincte ast fckt uma tnlokro douk numero vine we un mt pa. Ate 4) Arita of rieum am alogegobte dintre scwite mumere, exist donk a or dren st dit en 12. 1) Pemonseaie8 cea sal mick smd posblceor 4 mamere este pitrat pet. 45. Sis arte exist un mame deforma $21921,.2% divi ex 9213 40, Si.se arate ck print orcare 8t mumee din primeleo eu mumere nate nen, exist dont al zor raport ete o puter ahi 6 18 41. (Generatizare a probleme antericare) Fie np € N~ (0}1). $i ve orate ioncare ar f m-(p~1) +1 mumere aatrale diferite ales dintre primele np mere nasuralenenul,exité donk cor difreng ete o putere nena it 48. Se pot sooate dinto carers nist blocurl de pint de mase 400; 402; bt; 1B eu ajutorl a3 camioane fecare putind transport maxi 1600 kg? 49. Si we arate ch ovicre ar 66 mumore naturale lunte la intdnplare die lw primele 10 numere naturale nenule, vom gist prntreaoretea tel numer, I neapirst distinct, ostfl ince sume donk inte ele sf egal eu ltl, 0, Sk so arate exist m gn numer naturale ate ict 119)(2" ~ 2"), Generalize. 51, Alegem 67 de mumere naturale dinte prime 99 mumere naturale nenule, Sint printre acest dows care au produstl divinibil ev GP Bate 67 cama bun unde ales? (in sensu de ace mai mic, tn ans ca!) 582. Precupunen cf o chip de foal s mareat eel putin un go in cele 20 do Parte consentive pe eare lea juent. Dack a mareat in total 80 de gla fa fe 20 de paride jucate, demonstrat cf, intro anamité sacvent de paride onsecutive,echips a mareat exact 9 golu. 53. Ske arate ef aicare a f 241 numere prime mai marl ca 2 exist cl Pin ook care an diferent divine ou 2*** (K nude natural neta). 4, Fle muitimes A = {1,2,3,..98). ArBtat ch oriewm am alege 50 de ‘lemente ale Ii, exist prntre ele dou avand suma cub perfect. |S. Se consider 17 bile olorate. Si we arate ck Bi exit bile coorate diet ‘eae dons He sim prinre cele 17 bile 5 bile care a sown enlone. ‘80, Fiecare membra al unt grup de 17 savani corespondeaai en cia. fn ‘spondenta Tor este vorba unoa despre tel tame, Fiearepereche de savant ‘omspondeazi po osingurh tei. Skt demonsteae ck el pin ee sang conspondensi intr el po cern tem. (0.18) 7, (Tamera chinexd a retuior) Fie m gi dou numere natura prime Intre el Atuneiorivare ar dou numero intepia gi bexstl 2 © Z astfal Inc # = a mod m) siz = (mod 7). 458, Po un rind ge allt 12 seaune. Pe acesten se ngnsi 9 persoane, Ate ‘exit tre seme consecutive ocupate 59, Numerele 1; 2; 3; 3 10 sunt agzate intro orine aleatoare pe un eae Si-ve arate c& exist tre diate ee, dispuse consecutiv pe ere, care a sua eel pain 18, (00, ise arate cX exis un mami astural care aze ultimele pata cife 2010, cate eae dvi eu 2000. (01, Memb nel socinst intarnasionsle sunt din gave (i dite. Lista fnvinbrilor confine 978 mume, numecotae cu 1,28, 4078. SB 9 arate ex. Unicel putin ua member al acetal societal chr numkr de ordin este egal ‘@umuna numcclor de ordine «dot membri compat a i sau opal eu dublul ‘hmrull de ordine al win date competsot ski (O.LM.) 02. (0 variant a Principia fa Divehlet) Dacd'n bile sunt disuse in fl uel exists cot pain o cule care contine eal putin [g}+1 ble 3, Un yahist Joact cl puta 0 partis pe ai, dar col molt 10 paride pe AipbimAni Aviat ck dack josed suficant de mut timp, se poate gsi o serie tile consecutive pe parcursal core el jock exact 25 paride (64. ie 1 momore alow dintrepeimele 100 anmere naturale nenule. Swe te eX rinse acetone gage te are sunt lungize laturilor uni tonghi 105, Trorena Rnd Seekeres. $i ae arte orice sie de A+ 1 mumere rele ‘one un suse menoton format din b+ 1 termeni Fou weet reultat oo mai ban pos ach exist un gr de +4 cermon ‘ote confine un subir monoton de Iangime saat mace deckt +1? 166, Teanema tui Dilworth ( Generaizare a Taaremet ErdGoSchers). le injiv€ N° Si se arate aku gi-care ze mn +1 termes, exstd un subyit lcrasitor cate are hingianca mal mate dec my sau wn suber crestor da Tung dre doa Probleme prope 7. Fie n mune naturel nen, 2-41 mamererealo 0,2 interval 01} Demonstrate exit dot intel 3, i4iele=al 9, pont fice ‘agin a N persone ete mca ne atl enn ode ‘Sag naked mer 69. Un computer ete fot ting do 99 de ee int pend de 12a Sis tate ett dout le conte i ae computa» nti pi I ove 10, Gil compu women fe cna el inptnatie. Ct de mite conextn ente nessa as ela ve clulstoaretnprioante ti sigur oo de ite or te computers mcs o tnrinant soon vor pone © protien esemdndtoare: AGat! numivul minim de conexiun) pentru 8 foci oncomitent 10 competes a ispimant, dick ave Ib compute Imprimante eneraisare Mi. Si s0 arate ef oricare mame do 16 cle confine una san mai multe cite ovoreitive al elror produs este namie ptrat perfect. 72, Se consider o mnitime de ee C care confine gi pe 0. Si se determine wv multpty al In, me N* ~ {1}, eae e format din toate cle ii. 3, Fie A oamukime caro are-n ements. Alegem mai mult de jumitate din Folvonlinile hi A. Si ge arate eprint ncesten exit dou’ afl inca ne ote inclu i celal 14. Se considers un numi natural pear ete prin eu 10. Sk se verifier dack Pit o ptere loi p care se ermine DOT 70, Se cone un mums n> 3. Fm ol ma mare nando ma nc deci. wang I tmp pain nse Iwultinen {25 2, m), exis print el rot numero cu propretaten et unl Principiul cutie! In Algebra te yal eu sume elrtalte dou. ‘70. P(z) un polinom eu cofcentintzegl. Dac Pa) = P() = Ple) = 2, ne abe © Z diferite otare doud, of se arate cf mu existh d astfel inci Ma) TTT. Fie mumerele ay, a2, 4 intregi, Consierien toate dierenele oa, exalt wn moni i fc 6 al [8] < oe Frobieme Propure Pentru aceast totem Klaus Roth a pit Medabia Fels, cea mal mare Aistnege in matematic. NO. Fie 7 mamere rele 21, #255 27 SW se arate cb exit i,j = 1,7 aif 1 tute net alas fe iSamenraesy spas Sateen j = Tie aay 2 62, Si se arate ck ereare arf 17 mumere introgi, eit prize ce 5 care au un divin x 8. 83. Fie Ao slime format in now numere naturale nee, Hacare avin Civiocs primal el mult egal cu 5. $8 se arae ch exist priate acest dou, tare prods pStrat perfect. (G ponte arta chiar urmitorul reat: “Fie A malfine format din n+ 1 Innere inteegipoztve, nel unl dinte ele nesvind tn dvior prim aah mare decal mlea namie natural prim. Aritagi ef exist o submllime nevid WC A asta nest produsulelementelor li B si Be pitta perfect.) 4. Suma so sutd de numere rele este rro. Arita exis printe le 90 dd peech de cSte douk numere cage nu imma nenegntiva. Este act cel inal In rezutat pesiil? 185, Pie ay, 03,05 numece rene oareare, SE se arate ck exit un muir ral ‘oth Inede numerele +01, x-+ay #2-+ay sunt izaionae. Generalize. 1s, Pie tn name nataral mal mare deci 1. Demoastrti ci exist un num fin pgm gir infnit tit creveKtor de mumere naturale ay, 02) yy ase Inet toi tezmeni salut Do hay, phy + Ky, se prim 187. ve ay 9 49 A ry by os np de pert ab numerator naturale ‘lela 1 ho 40. Anitafte& pintreprodseleanby, aabs, , aba ald dou ‘Principiul cutie! in Algo care das aceag rest prin mpi ln 40, 88. Gist submited carnal 50 alo mols {2;2;..:99} ati incit ‘itm existe dood eemente pinto cele $0 ale fekretsabmulml cares iba ‘nua 9 sa 10. 80, Me tel gupui, in Secareafénduse cite 20 de personne, Siind e ‘ricare persoand ate exact 2 de cunostnte fa celealte dows grupuri, au se ie In conserare propril grup, sk se arate ck exist trl peroane, cite una din freare grup, eae e eanose it ele 00. Flee multime A format ea 181 namere naturale pirate penecte. Anta ‘exat o sebmuline asa § format en 19 elementa sum scestora ad aul tip de 19, 1, (Putnam 2006), Ariat ed orcaremulkime X = {215.29} dem elementa, foi 0 nubmulime nevids $a ll 2 shun elemant ftzegm ante Sac: n+ 54 a wT 92, 8k. 9o arate ck orca arf tel mumere prime, mai mas docs te, exist doula eltor sui sou diferent cot mnitphs de 12. (Putnat) 8, Intro clas sunt 33 declev care au sume vitor 430 ani, Bete adoirat ‘exist 20 print acta care au summa virstelor mak mare docs 2607 ‘94. Fie p un mumir prim. Atune,orcare ar fe € Z astfel incit 0 < a

1. Aton pentru oe neg exist donk numer ate {Fahy om ambled, ase act O'S fs = Yi OS fl S ghz = y (od * 6. Fie mulkimen A= (1,2,8,..,385}. Sise afl numieul minim do elemento care tebe liminate din aconsta, ate net 8 nu hina tre dstncte ct ropietan ck prods a donk dine ee egal ex al teien. Probleme prope 7. Aviat ef pentru orcaree > O exist un numie inom astfel ini Isanice for(S+a)} O exist o Infinitate de mumece n € N ute Sad Ik ~ Sconing} 0. Fie 12,285 8,18 21,9455, irl Ftonac, in care fcare termes, inegpind cu al trellen, este suma color ‘i ermeni anterior. Araftef eintzeprimit 0000000; termeni ai srl ‘fio putin unl eare se termini patra nero 100. Si se arate cl oricate a f 100 de numer Intrey exis 0 print acesten ‘citor sums este diviaibs en $0. (Olimpiad Rusa) 101. Pio o multime formatk din 5 element, Dintsesubsmifimileacestela Meem 8 submit ast edt oricae 3 ah allt un element comin Sh ve fale toate clo 8 submulfian aim element coma. (Analogic eu Problema 1. 102. SK se are eX cream am sloge 15 mamere dia girl 2,3,.,2009, oricare oni relativ prime, el putin wa dinte ele este num pie. 103. Sse arate ef mitimea numerelor rationale Q este dens in mimes merce rae R 104. Demonstrate urmitorul sr de namote fteeg P= 3,28-3.5 2 Principful entei fn Algobet one un subs innit astfel eet oricare dot termes le primi inte ei (orn 7) 105. Fie n numore intro positive relat prime, mal mari deci 1 ma mick lot (21). Arita eh printe seastea Se ai cel putin wna pz, 100. Se considerio malkime cu n > 6 elmuate gin +1 submaltimi distinete ale sal, eu cite 8 elemento fecare. Demonstrate priate acete aubinalink ‘vi ou care a exact tn element com. 107. Arita i orcare arf 11 numereintogidstnctecuprinse ize ~9 si , putem ii 3 priate le ere au summa nals. 108. Fie J = (0:1) sia = (0:1). Denim funetia fs J 7 asta 4(@) = + (1~a) pentru = © (Osa i f(2) =~ a penta = € (a). Arata ‘ovleate arf un interval J J, edt un namie m € Nantel Sac i AMI I este nevis, wade f (2) = f(FCoof 2), de mot 100, So dau numero naburale t,t ita. Avda ek exstk a, a ga, toate oul, dia muljinea {-150; 1} astfel ine sama : ats toate taste ‘fe multipin de 7 110, Sivearatec existé 0,8,e€ Za toate ne, satel neat al < 08 " lat bv2-+evil < 10" 111. feo mine don mamere este (yb) hum i atral nec 2". Sie arate hut pao subine $ at! nc indie sialon urmiarle ond "Col mul unl date ementl eee! pre Ge ere yw gt bmalinen 2, Suna taro emer iS ext dvbit eum. 112 Arai cl oviare ar fi nomial natural m exsth mumecle prime p i q sail inet n divide diferent % propuse 0 mick dist: ‘Teorema tut Dirichlet (Teorema progresiei aitmetice): Pie m gi n numere naturale nena prime fire ele. Atunciprogesia aritmetics bm mb Bom a ms confine 0 afiiate de terme primi. © primi versiune a acest tore a fost damonstrat de Dirichlet in 1897, ru cazul n sume prim, in saul urmaétor © demonstreazt pentru earul In caro m mu @ prim, pentru cain IB41 ek o generliece pent intregi Il use (into Iai Gass unt acele numer complexe caren i parien reall ston imaginart mumere inte’. Johann Car! Fiedvich Gaus, materaticin (peian, 30 Apriie 1777 ~ 23 Februaie 1855] Aportal algbris al acestel teoreme in teorin mumerelor consist in des yollacea analize’ armonios. ‘Ea este o generalzare a taoeines Ish Euclid care Aira ck muitimes nomereloe prime este infinité. [Eucla din Alexandsia(~ 100.Ci,), matematician yee, din timpurlecénd roe era Prolene). 0 generalizare a Teorema ul Dive Teorema Green-Tuo (demonstraté fn 2004): Eset progresi aritmetice or nae din umere prime, de ngime n, re N, mumds onan Aceast tore eto la rindal ein eax particulat al Conjectari lui Bhd atv a progres aritmetie 118. Sk.se arate ck exist o infinite de mumere prime de forma An —1, (Cas Particular al Teoremet lui Dircletpevind progres sitmetice). 114, ‘Sse arate ek exist o infinite de mumere prime de forma dn 1. (Ca ticular at Teoree lui Dirchotpuvind progres atetice), Asta ck oreare ar 8 srurile do mumere naturale (anno (ba)e2ts exist p# 9,2,9 © ast city > Oy p> by ty > Cp 16. _(Teorems lui Kronecker, [Leopold Kronecker, 1823-1801, matematician german). Piece numeral iraional-Aritai ef maltimes A= {yn-rnazynyn € ete dens in R. (Vesi Proema 109) a 8. Principiul cutiei in eometrie Fropricate 1: Fie un segment de lungime Pe acest ugent se af segments anni, Ft 1) Dac ty + ia +i > Catunel exit dou segmente care au el putin net con 2) Dock +154 ly <1 atunel exe un punct pe segment de angie are ni w al pe nit una dintze segmentle de bungie f= Ins 8) Dac ty fy. fq > Raton exist k++ 1 segmente pre cle n ent care at ct putin in pane commun, Propricate 2 Pie n Sigurt deat Sy, Ss,» incse intro Sigur de ale 1) Dac $+ S2-4..4.8, > atunel exist dou fig care au un pune 2) Dac 8) + S34 04 Sy < $atunc exists un pant pe surafaa de ale Scan © all pene una ‘ate supefeele de ace 8,4 = Tens 8) Dov + Sy +. Sy > AS atunel exits k-1 figuteare an un p Pruprietate 3: Pen eorpur de volume Vj, Vas Vs inluse ten eorp de volun V. 1) Dac Ve + Vesta Vy > W atinel exist dnd corpur cre a un punt tomo 2) Decl Vi + Vash Vy < V atune exit un ps ‘volum V care nu se afi in nll un dintre corpse de vl ‘fat in corp de Vui= Tn Principio cull fn Geometri 13) Diack VV) tou +¥a > AV tune ext 4-1 corpus ear an un pane 4.1 Probleme rezolvate 4. Fccare punct al unui plan ese colorat cu tna dinte cle dou culo, lb WW rom, pe care le avem la disponie. $8 se arate clexsts dud puncte alae la stan de tem eolorte la fe. Solute: Fic un trig echilateral de laturi un centimetra (vee figura). igure 3: Problema 1 Conform Principe cuie donk nse puncte vr ate aceaiclbare g Yor fla tanga do tem, Demonstrate eernativd: Pi A un punet ru. Dack unl dite puncte De cru decent gras 1 em este rye atc problema acl Ia onto, tte pantlecescull derek I cm, find abe, exst do inte Mana covth dena alae Obervatia Se poste attach exist dow puncte colorate deri ate I nc anf de cela Oeeraia 2 Se poste ait ek eit dou pute de aces clone alte ina dude dat, crear. Wien reformat ena ati Dracomenem planl in dou malfin dxjancte A gi B. $i se arte ed tin dntre cle ext dow pete ditnfa 1 om bi dow 13 puncte ast inci printre cricare tei dite asesten exist ware se ali a stan mal ini deet 1. Dovedii cd exist un cove de "one conse cal pati 7 dinte punetle date, Princip cutie! im Goometre in cazcontras, fle AB > 1. Considarim punctal C, unl dinte celle 1. puncte, Atunei AC <1 sau BC <1. Agedar, confore Princip cutie gas0 titre cle 11 puncte se aft la dstangk de mast 1 fai de nul dinte puncte “finan B (Be weet A). Atune! exist 7 pete in cereul de centr A gi asi 2 CGeneroliate: Se dow 2n— 1 puncte astfel inet printre oriare trek dintre cesea etd pereche eae se of Ta distanfa mai mick decit 1, Dove ct (Sinton cer de ro 1 care conine eal pin nite punctle dat. Presznten fn cele ce messi o problemi oarecum "eomplementars” cel Fem puncte fn plan cu propritten of ricare dow ft a 0 distant nk mare dc 1. 3s rate of aca ua cre care confine ote aoe pete tate mat more decét = She Fie un cue de rank R once confine puncele considerate. Vom conser tate crcl cu cette ace n puncte ide rai. Oreare dow. ‘Grenson enteroare dvares tang dine riare dou panete ete mai Sih doeit 1 Corel de ack R-+1 sm colin tnt soe w eter, Atwoe Tf cc rev oa ate deci uma supraftlr cern mi Bes (« +5) Putem rformula problem penta aru in care puncte afi fn spat ‘em puncte i sof eu proprictate c& inant dire orca dou ete nat are deat 1. Sk se orate eX raza orice see ere confine taste cele puncte in inferior este mai mare strict deat = ‘8. tn juteioul unui tionghi eeilaera de Latur 1 alk 5 pte. SH ate ckexnt donk puncle alltel dicta do oe molt) unul fat de ola Soluglo: Unind es tee segment miloacele lturlor tehnghintu ob ptr tung ebllaterae do Inara gun) Probleme reolvate Figura 32: Problema 3 Dost dintre puncte se vor afl in iuterioral unui ase de urmaze se vor afia leo distant de maxim ianghi 5 pin 4. inten plan se considers 5 puncte de coordonste intreg. Si se arate ‘visa un pc de eoornate ng ae bf ‘minat de douk dintre cele § puncte. " Nec ual goes sist ine sey cela, entre feo sua dow mere de eneag psitate ete pad, = illoc om egent are dept bait erdonatécmiouma i, respectiv, semisuma ordonatelor capetelor segmentului, at Obsertfie: Analog puem ade, folsindu-ne de priate cordonatelor, ci vir ar 9 pune de odo neg ope etd we eoordonatele fatregi. a mine 5. In jurul unei move rotund, ia dstang 6 is tnd, Is distant eae, se alk § personne. Flere lr mca cite oir care ret nie uno futuro cb 8 ns. Stind ef feeare peronnd sa agerat pe loeul grit, ak se arate ch ‘yolk mas. atl ne el putin dou erates ale pe cl Soll: Dana dao ne aga pf a hi al oral le me 2 an ot ponct,lae Poeul e sa'e ps akcan Principia eu n Goometeio a sonne I ncoeagi distant. Fi acosta d, Rotind masa in sens inves acelor de ‘eworie ot Toca bine eta 16. Lac table sah ve tale din doa coli diagonal opuse dow pris Bite posi i acopeiin supafaga de tabla rae x pede domino formate lind pte de maize egal ca exo al tebe? Solutie: Aritim cS mu este posibil Doul pitriele diagonal opase ale lable au acceasieuloare. Atane! tna dite cx dowk elon ale tablel apace Jin plus cu 2 fat de eoalatk Dar feete plesk de domino acopert exact oni pitrave de colori diferit. Coafore Principia extiet ma va exeta © ‘orespondengi de 1~ 1 intr cuorilepétratalor, del mu ¢ posibils scope. ‘TO tabla (e gem tablet do gab) do dimensiune 6X cate acoperith 18 ominous. $¥ se arate ek crice ampere a tales poate f impeith ex @ Tinie Ahropt asl init niet ux domino ma Be at, Soluyle: Tabla pitzett de dimensiua! 6X6 poate f mpl cu 6 tik nizotal gi 5 ini vortise; t total 10 Bait rept Tecate domino poste f fiat deo ingurk Ine inte cele 10 Presupunen ci exist ine diate oale 10 care divide un nami impar de ominous, Axadar intro parte a drepte se afl un mums par do pitzstelo provenite de In dominouriofnteogs din acea parte a drepte (cele ntiite de Jn), plus un mus impar de ptrageteprovnite, ete unl, de a dominosile Une” AcestTueru ste ask imposiil deoarece orice dreaptserzontll seu ‘vetinl impart cazeal in dou pi, fecare cu mums par de pltrle Aga ovie ni cle un muri parce dominour. ack Gecre line tale cel putin dovi dominouri, ecare domino find tii de osingurk line, tune tr her putin 20 dominou. Fas, ria uemare exit lini caren tient 8. Pomctete uni core se colores ea sogu gh mogra ast Ick ore tring ‘ellneralinseris neers abi exact dk viru colorate i ang. Sse arate ‘exit un pitt ines a cee cage are ol pin trl war colrate i ner, Solutio: Fis Ay As. Ag un dadocagon regula nsris in cee, ‘lunghiarle AyAcAg, ApAgAo, AvdrAn 8 AeAsAvy sunt echiatrale, loo 8 vhefurldin cle 2 tant eolorae in neg, ‘Consider ptratele Ay AgArAya, AaAsAaAus gl AndedeAva, Conform a ezobrate Prob Principia cutie’ veoutd eb una diate cele tei piteate are cal pin tel ie olorate in negra, See metre 3.2 Probleme propuse 1. In interior uni pet do ats 1 sin § punt. Sse arate exist buts le alae I cits url de ell de msi 2 in interior ums cub de atu 2 ahoaré 9 fur. S¥ ve arate ‘moment exit dot futur Ie distant cel mult V3 ‘8. Fle cnet puncte dierite in plan. Dum uneledintre segmente cae w Deve de ete dows pnete (au neapiva toate posible segmente). Sis a ‘hess dou puneta din care plac acelai ume de sogmnte, Gene 4. Colonia plan eu tre coor. Ariat cf exists dons puncte de ace ‘laze In dita 0 ici dicute: Nua gle care este cel mai mie numa de euler cs ‘tem color plana asteltnedt a4 nu existe doud puncte Ia dstanta la folorate. Act nsmér de culo se numese mural eromatic a plants, x Cert, conform acest exerci, 08 acest numiir este mai mare doc Deci xze ‘De asemenen se poate coloraplanal eu gepte eulor, prin acperie hezagon (esi fur), aol inet nu exist. doud puncte de cece euloare la 1 Figura 3.8: Problema 4 (a) 6 Probleme propwse it din gt rere ele gpt elo, bgimea tari hesagone tet ale convenab ( 1 aes, ct < Matematicinal maghiar Léselé Sadly a elise o acoperire cu ptrate @ sah, eolrndve eu sapte ely asl tne nu existe dou puncte la lotrel tao 1. vet fur) ‘Figura 4: Problems 4 (b) Ayador xsT Proilema determindri ezacte a it x rine desehisd. Ler care eae unoscul ete acesta 42 <7 rine cei care se ocupat de acest problema, considera ceatori aces N prolteme (pae-se & mod independent ural de cali, dap cm afin A ifr), emi pe legendaral Paul Eris, Martin Gardner, Hugo Huger, ine Hoary, Leo Moser, Bdward Nelson si William : Tite | Sh se arate cl orieum am calora punctele plata; eu dou eulori, va nt intotdeaina um tring echiateral cu toate vsfurile de sceeagi culate, i exist coloriri eu dou culo ale planulai pentru care nic un trivgh loteral de ltara 1 nu ae vel de evens cloae Se dan 8 poncte pe o ster. Sk se urate ok exist sense inchs care ane el putin dns ecee pact, Se dau 9 puncte peo seri. Si. ye acate exist dou Ia distant dle co Wn ‘Se dau suse puncte, orca te neolnire, Fieare douk puncte ge unese ‘ete pinto line regi fle prntvo lini abstr. Sh ye mentee exist ” SS Coometrio bn tang! monoea or, Ma mult, aritati ek exit douk triunghius 0. litium plan se due gate drepte neparaleleorcare dou. Si se arate ‘st dons care formeacd un ung de masien 30! 10. "Fe trol drepto care impart plato fn mai multe reghini. Sk se arate vicar am dstib 8 puncte in acest pln, nici unul ast pe yreuna a sept, exist dou alate ito scons egitae 11. Pe un care se nu 200 de puncte astfel ict mioura in grade a are format de oreace dou punete este exprinat& printrun mums ities, Se frat cl exit doa 12, Demonsteai ek Ul ascuit, puncte diametral opuse ‘m poigon convex mu poate aves dec cel mult te un 13, Laturle unui patrulater convex unt diametrclea patra disci. Demor ‘nent discur scoperd in intregime iaterioal pteulterult 14, Fle un pitrat de Ibu Consider 25 de puncte in interior Dlr, SK se are ob exist patru puncte printre acsten cate detonniod Datrulatr de aie ma mick dec 15. Pitedttete und Siw arne i exist doutpitricte vecne (ao laturt com) care coin tae de gah 8X8 se mumeroteantcu mumerele 1, 2,3, ‘mere a cor diferent este macin 5 4 16. Interiors ul ptat do lturé 1 se plsead un tung caren onting ‘entrlptratu. Sie arate cX una. din late tringhilal ee de onan 17. Interna wn cub de at 6 se consider 100% cubur una cu fle paralel c felecubutui dat, Sis demonstreze ct exists doud eutnes ante (0 proprietaten een 18. Se consid n forinatt ex ele patra : Demonstrate ex srl una se afi tn interior sa pe feel ctu. tn pitt $x latura de lungime 20. Fie AF maltimen itr ale li gi ex 1900 puncte artitrae intesiou lot stun tung cu vier nF gid nie el malt gal 38 tu jh: (Cole 2008 puncte aunt orizare 2 neolniare) Cit re le crt coats. Suma nner rr cle etal mick deat gine tee hee cl then dame ce rcpt eae ine pia dtc ek tpt. ate et pene son ol pt aves npr eos ak} 21, nioun ex de tu in 200 puncte. Ske arate ef exist inte ecten te panel cre snt scopes de alr de rah enn inser itr cer aie rs nak exter 3 sh 93. Fie un exe C de raak 1690 corns cicalaré A de ri exter fact intra 2. Ta atrirul ereulul C ave 650 puncte carecae. Si se arate ti putem poutina coroana cular atl foit ok acopere 10 inte punctsle fonsiderate. (Martin Gardner) 4. Siw arate ei exist « mom natural emul, a $ 24 stfel int tana < 3 (siian Preda) de format peat A se St 25. Pe o supra de frm’ pitrad, de are J, considera NW pune elle 2 ain ind ef, oScum a pasate ele pute, exis ol pain 3 inte ele cae forneagi'un trig de atl mal mic sa exalt en 20 on 26, Fie un poiadru, Si se aate ck exlstk dou fog ale sale care aa acl wie de lat 27. Penctslo wn pan sun orate xu ums ft de cbr. fn acat plan ie de dn doa panes diate O a A. Pent care pant X din plo, die ©, notin ex a(X), aX) (0), mira ard ung format de OA ‘OX, mist en pom. Pe CCX) cre de ze O rsh Se Tgte OX-+ 38 Aral chet un pune ¥ atl net cereal C(Y) ae tm pct de acre lomo ¥, (OM. 1984) Principiul eutiel in Geometric 28. Toate virfurile una poligon convex eu m lari sunt puncte laiciae Dunno dak i ntesonl Se a al pact atl 29, Un pltrat de laturi'n ext format dia 2? pitritele, fecare colast cu ro slben san verde. $i determine n minim asf ine, penta orice coor ‘it eiste 0 Tinie go eoload cu eal putin trl trate unitate coloate {acownsiculoae spe lini clean}, (O.N.M, 2006) 80, Ficearepunet al unl plan este colorat ca rogu, verde se albus, An ‘exist un dreptonghi care ave toate vifurlle elorte en sexe! cilore 82, Unele arco ale una cere sunt colorae stele: sun ngimilor lor epigeyte un stor ia langimen cereuli, SK se arate ch exit un dept Sseris i acest cox care are vsfuril neodlorate 88, 12% din aia unei ser este olorati. $5 ve arate of exits un pled inserie fa cea fr cate ane vitdulenocolorate, 4. Uncle atc ale unui cere sunt colorte. Suma lngimilortuturor arel ‘oleate ests mal mick dect un sfrt ein lngimen cere. Arita exit ‘un pitt inser in cre eae are tone wifurilenecolorte (elician Prods) ‘85, 12% din ari une sfere ese olorati. SK se arate ob exe un cub inser In acest fr are are vaste necolore, 36, nte-un dreptungi de dimensiuni 20, rxpoctv 25 unit plastm 120 de prado dimensiune 1. 8 ee arate eX putem aga un cee de amt 1 in Imerioral drepiunghiatl cares ee intersects eu nil an ptr. tc don dimen pa in © pant, Sk arate exist dou puncte alate la distant de maxiat V5. ‘38, Pie un patraleter convex de ase §¢ porimetra P. Si.se arate ciputem, 0 _Probleme propane lass un ee de rash ff interior actu patrunter. 189, Fie 80 de cca cu Tibi agente pe o mas Se sie ck tonte cenurle indiok ora exasté. Sse arate cK exis un moment In eaze sume dstantlor de Incmteul msi la centre ceasriloe este mai mi decit surna distanelor de ln contr meee! In wirfurile minute 40, emeatle um tel cum Hi im cloane st 0 Taba te Stnpletat atl nt da antonella w a in ‘Secu manera Ge inh close rept ets el palin m5 0 Ste umn tutor elameneoe abel ete pti 41, Seda dows discuri unl mai mace dct exit. Flocar disc este diva In00 de ouctoae eougruente. Fieare sectoral diaculul mare st coloratarbi- tur fe eu rog fie eu albatru (o eve snetoare rg go aust sectare albus). Disc mie ate sectoaree eolrate fe eu roqu, ecu albastra, mu neaprat tn ‘nmr egal. Diseal mie est plat peste diel mare astfel int centzles se Aoatele color douk dear coined. Sk se arate ch exist o paiiotn care mums fectoaelor dlculus mic care an acces culoare eu sectautele corespunzstoare I discal mae ete de el pun oeut& 42, Visfurile na poigon cu 50 latur sunt colorate 25 ox all gf 25 cu neg Sse arate c& exist un vie care are asbelevifuri vine eoorate al, 43. Un cit ote imp in 100 da sctoare de cere eye, Necare find cola fie alb, feo negra. Shad e& 60 dintre ele sunt enlorate ou neg sh 0 Inte eh cist seven de 40 da setonre consecutive eazeeon{ine exact 24 Ainsetonre eolorate ev Deer 444. Fie 10 puncte in interiorul unui core de diametru 5. Si se arate ck exit Uo panete printe acest alate Ta distanfa cel mult 2 45. fa interior uo pitt de laturé 1 eo aft mai mute corer ear an fs tangiilor 7. Si we arate ck eit droape car 5 ntrsocteze ee pain inte acest ceca 46, in inerioru cercull do ra 6 ge af 20 de sogmente de lunge 1. Si ve fine e se poate duce o dreapth paraei sa perpendicular peo dreape dah a Princip cutie. care intersectonn a putin oud sngmente, 47, Fie 50 de puncte in plan, orca tri necolniave. Aosste puncte Gora, fccare et una din patra cull dste. Demonstral ek existh. 0c {jd pufin 130 de trunghiae sealene, ca vifurle rine punctelecolorate ecastt enlace. (OB.MA) 48, Punetele unui cere se coloreaat cu verde sau galben atfel inet ox 49. Fie don pentagoane AiAyAaAeAs si BsBaBs BB. Dacem tonve Thntele ee forma By, j,j=175- Calm cu rogu i verde toate cele 35 foment ale fguri atl fot sf mu se formeze ne un tringhi manocram Bi'wo orate ci tate cele 10 muhi le pentagoaneor sunt colorate cu sce cloare (O-LM. 1979) ‘50. in intriorul unui exb de laturk @ 9e considers 1980 de puncte. Si ‘emonseae ef exist do ex propietateac& distanga dnt ele este mal ‘ect 1 G1. Fie punctele Ay, Ayo) Aay defini ecursiv in flo urmitor: tro AdiArAy oss clara, puncicle Aas, Aseas, Aaays sunt mibocsle turilor tiungbinil AAsa-ada,-rdan, entra feeare KEN, & = Trn= ‘Reese 3 puts sunt colorate cu dovd eulori. ArBiai ci dack > 7 a vst eo putin un tapes snes eu var clorate eu aceeas culoare 152, Fle A, B,C, D, E, F gas puncte in plan, in porte genera ricare puncte nocolniare). Oricaretritmghi format eu aoeste puncte are laturile Ini derive. Sse arate ck exit un segment determina de aceste pun fave ets em ma ich Intra no ung gt totodask cea mal mare laud ‘td tring a 4. Solutii probleme algebra 1. Conform Principals ented exist doutdintre cele 12 numere a eitor Uileeats care este un sume nema mal mie decit 99, ete un mulip de 1, hc de Ferma a 4. Oricepiteatperfoct se termini int-une dite elfoe{0,1,8,5,6,0}- Com an 6 cut, facare purtnd deeptetichatk una date ef maltimil a Verioae. Ficcare pltst perfoct, date cele T considerate (aceste mamere sunt ise) i cise intr-ana dine ext. Conform Principal ext exist cel un dou mumere n acess cuties deci dteenta lor est divibilé eu 10. 8, Deck exists el putin dows parzoane care nu au allo cunogtins prntee fel prozengi, atuns cele donk persoune au acelasi muir de cunetingePre- Fupanem c& o singura persoand ave 0 cunogtnge. Atune! cdelalte 14 su ite 1113 cunestinte doc, conform Princip cui exist douk care au acelagh nic de canogtinge,' Dack feare ere cel pug o cunostint® tunel Becare round dinte cele 15 are un mumtr de cunostinge cuprins Inte 1 $1 3 nln Principals etn, exist dow caren aclag nuke de eunostnge, Oserooie: Enanful este valabil pentru un numdr earecare de n persoane 4. Pvideat suntem in conde probleme! anterioare. Relasia de 'a cunoaste nth persoans = reais ja 0 parti eu 0 alts ahi 8, ait 16 poset diferte de nile le numer (A-AA-B.A- (is.nd.)- find poste 4 intiale penteu mune gi 4 penta prenumne, pe de 0 te 17 peroane, po de lek pare; de unde concusia. Grice namie natura dient Ia putaren a -ase termini nte-una din eee 11,56 doc), conform Prineiptul ule, exit dou dintze cele cin mumere Solutit probleme algebrt a ‘cate au puteres a 4a terminal fn acco ifr (Patemn conclusions. i deren lr este mail de 10), 7. 6) Thicoloral, st el omnes cel moldovenese, are tei culo ( find de oigine ances). Find extrase 4 bil avd tel ealer posible, form Princpilu cui, douk inte bile au song clonre ) Dac extragem 13 bile i aver 3 eulri posible (agadar act extragem 22 bile stun pot f print ele edte 4 din fecare ‘Rspuns; mum mini oerut cate 12. 8, Imply mumeree te 1 20 ace maim {1,11}, 2,12}, (10, Continue tn aceag tod fmpdfiea cu amerele din inervlle fi, 0} 0}, {61 80, (61, 100). Avem in total 50 de malin. Dee dae slegean 9a ‘umere vor exist douk din aeseai muline (cae au dfrenja 10), Pentru dient 12 vor constr mulinile(1, 18), (2,14)... (22,24). face la fel petra calelalte mumere. La strié mal sknda mune (07), {08}, (99), (200). fn final vom avea 52 de mili. Dac legen 55 de ‘vor este out din acoeas mulfime,adicd vor tren diferente Ta Pater gel $5 clemente asi ines orieare doul itty le au di liferits de 11 Dim un conttawzamplis (152504152824 1385455 69 0 577380;-.99) 9. Constrim mull: {1, 2), (34), Qn = 1,20 <2). Vom avea total n 1 mimi, Alegind n wimere, vor existe dou eae spain, ‘multi, Suma aessor douk numere este lpr, 10. Dai ot 29 clvi (cel $0 eX Stefan) au Bicut ficare ced malt 14 (ogndar inte Os 1) existh 8 care at fSeut acta: mum de ese, AL. | Considerim malkimile (1:2); (8): ; {803100}. Pind 60 de mali td alm 1 de numer exist, confrm Principia eat, dod ume ‘wr din oceeagi ulin; deci consacutive. Dar dou numer coasecutive Dime inte ele, le unde eoneluia Obvervatie: Daci nm ales 60 de mumere na a8 6 exist oliigatoria do ‘ate fe prime nee ele. De examin am fl puta lua merle {2s ty 00) 12, Fle mmercle 245 2! = 4, = 1. Aoaste a+ 1 mumere dau a “ bem algebras restr prin impltre laa. Dar exsth « posible restr dferte,agada exist ‘oui peinte ele cae da acelasi rst pein imple la a. Presapamem eT W2"—1 dau acetal rest, unde c,d = a, ¢< a. Atunel e divide diferenja lot (2-1) (21) = 24@*— 1). Dar ag 2 aunt pte inte ele dae no c= beonehiaonitn, 18. Consderém wrmitoarele sume 0 a3 +09; a3 + 02+. + ayy: Da inte ele una din sume este divisbi eu 10 atunei derionstratia cheat. Presupanem cl niet una din sume na diviaibils cu 10. Ate exist douk Wot ele azo dan acslag rest prin imple Ia. 10 gia, prin mare, diferent Wvibié cw 10. Dar diereata esto tocmal o sui de elemento consecutive, dar cevings. 14. Cousiderdin submuinlle {2}99}; {2:98}; (49:51); (60) in numdr de 40. Din fecare mitime aver wn clement Si mui unl fu mulimea A (dc fn aves dou din acest muitine, ar exist don elements ex sma 100) deck mn sun element din submulimea (86:64), deel um pte perfec n 6. I. Observien eX acste tel probleme sunt practic echivaleute, Vom rezolva oblems 17 Vieeare punet este capt cine segment Yo wn punet. Cam ele capt pears cine segmente (till) si avem douk ev: Iori (cus), rt dine segmentae care pornese din A an cota clout. Pa A restrange generalitatza puter presupane of scestesogmente ant AD, AC, AD, sou roi. Dac nl dn sementele BC, BD sau CD (de exp BC) Ihe eogo,atuncierumghisl ABC este color rogy, in eaz contr tiunghitl DCD este ab, 1%, Aver 500 de cut. Preenpunem cf exit maxim dou eu care conta mir eal de bilo. Cun tn fete sunt anim 240d il (des 241 posi) iri 8 le mas 482 ent Dar ele sunt 500 gi reals eontradictn, 19. a) Cum am scos 4 tile sunt tre enor trebuie ea minim douk sib i elon W)oxs+i=16 20. La impiryjrea cu 7 6 wnat numie rezule resturle 012,456. Pitratal in vo da la parties eu 7 una din vesturle 0.12.4. Avem eines mimere gh patruresturi, Cel putin dou in ene cine trate dau acl! rst ampere ou, Bie lex sty. Atunct 7? ~y) sau T(z-+y)(2—p), det concluin, 21. Numiral de subgrupe (eubmultimi) ale mul A formate din cele 10 lemente este 21°. Nu se poate ca una dintre submultimi si fie chia A pen 4, im acest ca, cealaith mule ar rebuke tot (pentru a aven ‘uni i cle doud mu ar & distincte. De asemenes exciudem submulines vid in urmare muméral de submuljini difrite pe care le avem in vedere i 2'm 1022, Suma elementelor oleteleubmailial » hk A este cup Intro 10 g 90+ 08-401 = 858. Avand 1022 posiile diferteeubsoulfid ‘855 ponibile cue recut c& exit el patin dow submaltinieare au ‘um. 22, Proodimn ca tn problema anterour, gistin dou mull nevide i tinct cco 58 ab aceoas umd minis eamentele omane. In moet fl ‘un din submalgini nu devine vid (a poate f nelasi ann in colnta pent ‘ou sumele egal) in cle dou sunt dsjunte (am eliminst partea comin Result coneloia, 23, Cole opie clemente ale multinil $ pot forma 27 ~ 2 = 126 subsmul ‘aclu mais vidk i pe ind, Suma eementlor Hcl malin ‘cuprinsh tee Lg 244284. 19=129, deck exist dow submit etc sunt (pot ft cblardsjunete dae dupa oe lesim gist eliminim elerentele mune) 24, In malo considertt cast 5 uomoreimpare. Asedar print numeral 4, any ts Ty 29 a¥em 10 numee immpare g 8 numere pa. Cx 9 ai produit (a1 1)(¢p ~ 2)nfag~ 1) contin fcsare ete Goud elemente Cele 18 dec), conform Princip ele cel puna dn fate format di ‘oui numte ipa, asada pas gi de! prods e pa. 25. Cu in excita anterior avem mal multe mamerempare dec pare. Del nu vom putes face pereei "dasa = seastor” de mumere de pit diterite Prin uemaee unul dire factori va diferent de mumere de acca part ‘paar prods va fi par. 26, xtrngem 13 ereionne, Conse cole 13 esate dept "biota! 6 Solu probleme algo ‘ale 4 euloridrept “cut. Conform Principal cuit oel putin 4 erioane tnt de aceesicoloare. S& demonstri ok m= 13 este mune minim. Dac fun exteage dos 12 creiane, ar putea fi doar eéte tri din eeare dint ele 4 ‘loti Mentonim e& aceastsituatie nefavorabil ete posbiltdoonrce exist ‘el putin 3 eeoane din fare euloae 27, Fie maltimile A; = (29(2i—1}\p = 0:1.) Acestea contin mmerdle Inypreitmpreun ex multi lor ma ie as egal cv 20. Malimen A, cone ‘tea numse izapar in mulgimes 1;2:.2n impreuni ca mulipi Vor fn alin nevide gi dsjuncte (sunt n numer impare) 8 cor reuniune este (1:25.20). Dine cel n++1intreg dt vor fin aceeai mulgime, aniline care scl puta dous elamente, unl impor si dublul ek, 28, Ani on +1 mu ete un num scent de mare, Fie mulls Ue Gb I 5 2k+ 1 formaci din b-+1 elements. Suma oveivor donk numere difritedinte aeaton foie cuprns tne 2k-+ 8 si Ak-+1, iar diferent oriinordowk aumere dierite tute curing inte 1 ik. Agedar nil o sum inc a difrengi nu ote divi (24-+ 1; mumirel tat este cel putin k= 2 ie A‘o mole format din k-+2 nuzere inte *'Dack eristh dou aumere din A care dat actas! rest prin impirfire la 1, atine ferent lor ete divinibia eu 2k +1 In eaz contar construim urmétoarcle # + 1 submltimis {0}; {1;2K); (22k 1); -: (Esk-+1). Impictind cele k-+ 2 nunere la 2 + 1 yom obtine 1) rest! difrita, dee vor exsta dk care sunt intr-ana din subumifile tomiderate, dei vor even sums mulipla de 2k 1 th 20. Vor impli malinea mameaor (241710, 100} im wemitoare 18 ui: (1); 4100}: 87} (4 88} 82) Find 18 ii eal) 19 name (ie) sige vr exita dott mimere in esos line (vient “ince mime de 2 elements acrsteaavind sma 108 30. Fie A +1} molimen format cu cle n-+1 mumere naturals istincte lose, mai ici decit 2n. Presapnnem deasemenca ch el sunt endo hate in ordine exesctoare, tunel B = (on41 ~ 04;0y41 = dajty4y ~ On} ene o mime format eu numere ‘coral distinct thre ole ml tel deck 2, fa toa, in eee dou ulin, “r Sol probleme alot vem 2n + 1 numere naturale mai mie doce 2n g conform Principia cutie shui dintse ele (ete unl din fare multime ) vor egale. Agadar exit He (Ue asmbT} sj € (1 2nsn) astll nest e = aq a5. DO agin = as ‘BL. Proveimn en in problema precadentS. Constatin of dack algem n+ Lie din cle 2n + 1, vor exist priate ele tre atl int smn a dou es Be egal ca alte. [Numirul maim cout este n+ dooarece putem alee, conform conde di a, sumerle{1;8:5:.-52n-4 1) 482, Presupanem am sles dj ceo n-+2 numer. Dac pare le m0 ‘mun atone emi pe toate ele lee atl fat el mn tare. tle devink 3 (frend face numer rim sels pte) A In nat moment dont posits 1) Princo mine ase a aft una inte numerele n+ 1)-+2-28— 1 In cst cr diferent ints On 9 musa repeat ese cup inten 2) In en contrar grup aumerl celal in n+ 1 erect: (0:20), (2.2m Xjo(t3n~ 1). Find n grape plas oumirul Send dea fe a 4 ‘minare sigur dou dintee el vor fin acess graph wor sve aeenia Intro (3). 88. Se partitionoat interval (1;2") in intrvale: (2): ,2) 5% 2%); AY) Confarm Principia cute, exists un interval date cele considerate cae ‘nbn cel atin tei dntre cola 2n-+ 1 mamere. So observ gosto eel ne ‘were, eypringe inten asta do intr, sunt Mogi lator nal rung (uml ado nee ee ele mii, est nat mare dest al teil) 4, —Numisultipuror de marion exe $-4-18. Agadar penta aves 11 idontet vem nevoie de 3-4-18-10'+ 1= 216) marie 35. Considerim subsmliinea Ii $eate nu contine nurse divisiite en 3, N= (15705 30m} 4“ Solu probleme ngs le cardinal Sm, Notim elemental li, seis in ardine resctoare, Consider mitoarale submit ale al: (38; k EN}, Ay = (0,38; FEN), oy Aa = (a,34; KEN), act £ > 8m-+1 atunc cel putin uns dinte submulin contin cl putin lous element ear, evident, se divid woul pe cell, raportul lor este chiar pvtere nen lu 2. Fie gals axtfl inet a, 3% < 30m < a, 34 5 by = a, ¢ = TF. Aver 1m < By < 30m. Mlgimes {2; fa bam} confine 8m clemente din $ care sw propretaten 0 < <3, oricare jj. Agtdar eum bo mpar st mumerele 4, sunt difriteorcare douk obtinem $ ¢ 2. Dect mini lui eate in 1, (Wei Problems 27), 36. Tmpictin mumerslo in urnitorele cutis (18),(2;7).(86)}, (49). Ovieum am alge cin numere naturale vor existe dou in acseast mukive, lock vor avea sua mal mare dee § ‘7. Not eu a aumdral total de crt eumpssate tn primele é ile. Conform ‘ountulai vom avea: Tsay 3 ponete in plan, oricare tre naceliniare, Si se arate st cal mult n(vt~ 1} teanghine ccc gi ecilterate cave an value print aceste puncte. Demonstraie: Pe mdiatonen unl segment pot fi masien dou punts (in ‘xz contrar arf to punta ela), Deel Geearesegmnt poate H baz ‘ol mult dout teiunghit lionel a ebllaterale. Cam n puncte frmona ‘Solufil probleme algebrit C2 = 5-0 aaymente, in total vor putea fool mule 2+ MAM = nln — 2) Uulunghinrt iosoele eu echlaterale, Cum n puncte, odcare tre necobinire, sunt virile a C= MO=tXo= \viunghiui, inseam cee sun vicar » eal palin =nla-1) trunhine ccaone. Rovenim Ia probleme noastri. Find 60 puncte gid conform Princpila cute, vor eel putin 13 la fl colorate. Cele 19 pane vor forms cel pin 1303-03-9) 4 20 rho selene eu wisurle colorate eu acoeas culoare 40, enum sof care adresens treble A si pe sola sn eu B. Cel late perechi vor & denumite C— D,B~ FG H,! ~ J. Stim ek B,C,D,P, F,G,H,1,J au dat mina eu an mame diferit de perwane, ma Matin wun (5 2535 8) ‘Dac sofa cau care inzeaba arf dat mina eu opt persoane atc a 8 practic eu (oate elllte parsaane (sft se exclude conform ipotes) $i el una dintre elelataperooane arf dat rhepunrul zero (petra obo & ‘mina ce putin eu dousana 3) Fie C persoans care & dat’ mina cu ali opt. Acrsta a dat mins ca xa scp pop paren. Aguiar repel sto font dat de partencral sta Presepunm acum c& B a dat mana a7 perroane (ac soful mu eu D ‘dat mina en seo persane). Deci psoaaele E, F,G,Hl,[,J au dat mina ‘ol pin dows pesoane (cu B eC lecare)- Agedar maar exist nc ps ‘nt sf dat mana de dous or Fle personna care a dat mana de Tort. G22, estat cu minim dowd pence, Proerind analog reeulta ea peredle GH gi — J eau salitat ox 6312 per= sean respaciv3¥3 pone. Hine ci fac ae ase intele ‘ew salutat eu penoane. Ober In oie poland erin ot 4 persone, sia ‘salutut o singurl persoani. “ __Solutlt probleme algebrit 40, Dac printe nomerela 2! ~ 1,22 — 1, ay 281, mex nil unl ere ‘Ui ros O prin imparyre Isa atunc vor exis dow 2° 19h 2" — 1,2 ce. Hpetind acest rtionament daducem oX x gy au aclesicifre. Agadar avem 2 numero care da reetaridiferite prin imple Ia 2" 2° restan posbe, Daducom ea urmare existonja unui care se divide eu 2", fe acesta wu Deci ifm =p), sp 42, Fle numerele 0, of, of + elvan f +0 +. +0f. Exist dou peinie ferte n+ numer cote dat col est pai Impérte len. Atune fren Tor ste divinbls ou m. De ic, prin renotare, eau ceriata 43, Dackdifernga doe dou numere naturale fre ete m stunt pot thea dvr comann nom pri el ml egal x inc cle 10 mere naturale conssatve considerate, cl mult para vor 8 tive cu $ (dou pare, dos imate), eet mut doa wor f vba 3 {ul par, al pa), cl malt douk vil co 7 (aon pan lel imper). (im piste ele 10 mumere consecutive considerate 5 or inate cel mul Jotea vor avea una dite divizei prin impai8,5, 7, va exist url care va Ain eu edeae mer 44, 9) Dac primal name par qi suma dint primal tal doen eve pal ‘net gf eal deal delon mami ete por. Aste toate cle 84 de numere sunt fe Foc prim par, atune tate sunt ispace Noe 7 dire el."le pot da prin impr la 12, fc wnul dint resturile 1121468410, dc suut coat pare, fe 13,5,79,11 dach sunt toute impare. In ticaredntieeuaari donk dite mere vor da noelag rest prin pila 12, ‘wea diferent for vf dvb eu 12 1) Cole mai miei aumere naturale nena psibile wr 81,35. 2834 Sha lor este 83 do unde coneluzin. 17 45, Considerin veraMontele 9214 de numero: 321; $2321; 821924521505 ee 321921.321 a) 8214-4 muir avid in component de $214 ori gruparen 3 Prin mpiyiea acestr numere la 3213 9 vor obtine conform Principals cel putin dova restar opal. Send cele dou mere wom gia tn tunis forma 321..2100..00 divinibil eu 3213. Cum 8218 ete prim cu 10, ccrint. 46. Yor considera uemitoarele 8D submaimi: Ay = {SMijk € Nyé ‘x 5,1 5 09}. Prinite cole 81 aumere considerate od! putin douk date ok ‘tn acseasi mukime, agar raportel lor a flo tere «at 3 eu enpoasat 4%. Oriacodintre cole n-(p1)-+1 numere ete de forma nt-a, nde a in eet (p~1) valor mat miei dest mp find prim en n. Guan obliga ot dine umere vor deride cea fom apo lor va Boat < eg clo gute sumer natu ale, fa ewstoc, dsr ple 10 manereoatrale mele, Net een a bast se miner Not cD tine hema cu tuner a sere inte dy ~005~ 15-14-01 Cdn AUB oe 0 (tote sel sta i egal 0) tap ce alge fe impresses. Agar ntrefa clr dea mal cae aca ‘Prin urate exis Fp atl not ox = op ~,p nfind obligate dl 50, Consider mumeee nail 22:2. Caf Pri 2%; 28, Conor Principle ‘eth dou pie ol cre dau acl et prin tpl la 1 an urmae diferenta divizibil& ew 119, me * Goer: Se conser amen ab Atal ei 2 mumece naturale astiel neat al{b* — b¥), q 51, Pine pina 8 de sumer tr els te 3 ae wnt de Yi 3 Ata pire Os mene dntcs 2 Dee eer ut pr. Pria we eee deseo ea ‘care ou produsul diviaibil eu 6, “7 se __Solujtl probleme algebra (67 ete ool mai tun numie pentru cin ca contra an puto an 6 de mere fare nu Indeplinese proprietatea cera sanume pe I; 2:45; 97598 (adic po fle novia eu 3). 82. Veui Problema $1. 43. Colo 2 41 mumere prime, ind mai mari dee 2, cunt toate impure gh se dau ca rest prin impels 2" yal dintre cele 2 elemente ale mill: {,5,5y.2"**=1), Cum in total sunt 2-41 mamere, vor exist dou care da esl et prin impirsire la 21, Difrenta lor eae wn mums diviibil eu 2°", 4. Consideram mnitinile {98:27}, {9728}, .., {68:62}, {61:3}, {604}, oy 98,26}. Suna elemeatelor din focaro multe este fe 125, fe 64, deel ev Jrsfot. De asemenes mai consderitn msltimile (2) st {2}. Sunt total 50 de ulin. Com tcbaie x slegem 50 nmere vom area urmésosrele variate: J) Hx, printze esl 50, dou mamere din accel mle, agadarconclsi 4) Na exists dout elemento din nonnsh multime si deci printre cele 50 ge afk 112 procam gf unul diate elementele malimit (85,02). Atonc fe 1462, fe 0 est cub perf 6. Dack arom maxim patra clo ex care sunt colorte ene 17 bile atc, fonlorm Principal cutie, exists cine! bile de acess culonre, agadar 0 wae ft a copcuse- Dae avem cel putin cnet exlort reali cena variant fonelaie, Geveraizare: Pe X o colic den obicte (n> 1), nu neapirat datince. acim = 0? +1, cu nude iniey nenegti, ardtai dare oe ce pin wt tin urmitoarle cara 1) cl pin a-+ 1 obiecte sunt identice 1) el pu e +1 obicte sunt ditncte dou efte dows (Un eaunechivalent pentru probleme 15, 16, 17 eae urmitora "we persoaueeorespondeast pe dous teme. Oriere dost persoanecorespon- ‘hat pe osingurk tena. Atuncl exit teh percane care eomepondeesX po wee tor” Novem In problema noastrit. Considerdim un savant oarecare dite et 17 Vic aeesta A, Conform Principtult cuit vor eista gue alli saan ecto ‘etn ea coresponda poo ncoegl tmnt. Duck dol ditt scotia corepondenrh Inte ei pe tame respectiva, problema ete inchcitt. In exe contra ce ga ‘van corespondeazit inte ope calle dous tere si contort enna dr Solutt probleme algebrit smal sus, rut contain, PPutom extinde asta "Riecare membra al unui grup de $0 savangi corsepondessd cn cela corespondenta lor este voxba numa despe patra tems. Fiecare pereche strani corespondeazi pe o singur mak - Sk se demonstree cb cel puta, ‘svanfcorespndesak Ince ef pe acess tem." 57. Cossderio urmitoarele n mumaere 2,04 moe (=H Fiecae dintr elo este congriea oa mdilo m. Aric nu exist print dui ares fe congruente modulo. Prin redocere la absurd prespaner atimse+jm (mot n) Aceata implies, (ym (045m) — (a+ jm) ‘ste diviaiil eu n. Dar min su prize inte el, iar f— j| < cont ‘Agudar cele numere considerate ds ret diferite prin imine lan ‘va existaunalprintre ele, fe aela a+, care den alas rest ca fb Impcie la, Numa aitat esta z= a+ 58. _mpirtim cole 12 aesnne in patru grape de cite 3 seaune conc Cum se sgeai 9 persoane pe 12 seaune vor 3 sane bere. Ayar va ‘un grup de tel semune ocupate toate ‘Observe: Dac arf doar apt persone, conchusia tu ar mai sdevieath ‘ian Ubere al tells, al suse, al noatlea gt rgpertly al doisrevoclea so 59, Procediim asf: Lishm deoparte pe 1 gb porn fatr-umul din (6 zcem in sensu rigonometrie) le impagin pe eslellte now ner in srupe. Dack fucazegrups are sua maim 17 ste) eutoa taturoreelor fr maxim: 1-43-17 = 52. Contradcie fap ck sua tvturor este Deci exit toei mmere consecutive pe cere care sma cel putin 18, 60, 128 1, Membrilor color gas Yet corespund aso eubmlin ale mul {1,209 078, mui care sunt dune gt chtoe rnin este (1,2, u Solu probleme ager uta celegasesubsmultinl represintl o parti. Conform Principal ewe ine datre muikimi, te aceasia A, vm contine cel putin 330 element, Fla fi b+ 1 atonet saa ‘oto un subsit movoton crit de longi cel pin FF. ‘Alte, daca (a) F ovicare ar 81 <1 < 41, proedim ast: ‘Avorn 2 +1 suicide lungime eal pia 1 (nevide ea mult, Vor fl pin 1 absurd ine yin. ing ere an acweagingime nt if ‘tune a, 24) 2-2 Biya Tralee, dace m sin antl neat a, My hore Ia subyira nial care env creteStor, de tangme maxim are incepe fej, putem add la stinga cel putin un element $i anume pe a 3 pin imate cost eu eal putin un Iungtnen cuba Anal, dct ag > ay tunel m- m. Si determine mumiral minim de legiturlealewator = imprimant’ safe Tc, dlocd nun moment date necesar cn caleulatoare sre simulta In pei rants, acest here pribil Solute « Generaiisér: Trebuie ca la fecare imprimantS si se serie atune lind n calcultcareenrecare Te acceveazA sultan. Atte! 0 imprimantd are: fee are cel patin m—"n +1 legitrt (duct ar avea cel malt ms nc cele mcucalatoace la care arf legal nu ar pte imprina smnitan) Drone ave me imprimante, aumiral minit de legatur va n(n 1). 1. Fle numaral de 16 cifte Gdj-dhg- Dank una dint ctr ete 0,1, 8 stonct problema ete ache Presupunem ef clrele memtrut sunt Imuljines {2:9:5;6;7;8). Atones Scare citd sau prods de cle are tn ‘ompunete numa factor prt 2.85.7. Considerdin uradtoeree 16 m = dita = dado = dydy- oe dye. Evident 2 este de fornia 2°59 Pentru focare ¢ = 1:16. Flecare dine nameree my by ca e, Iimpar. Agdar asem 2 mere, a pai ‘exponent pari, dei va fi ptrat perfect In cazcontar, dou dine manera, ences 2 gi 2, vor avea acseasidsteibutie a pair. ria mate, dao 2 > ay ates 3 = dyyyeudy ae in descompnerea in etn! pei tana ‘ponent pari, dase pitrat perfect. 2. (Vest problema realli nz. 7 problema 45.) Flo (evies.;eu) mulimea etl dat priate ate se a8 $0; He cy, (Construan mma A = exer Fie acum seu de numere: ti Ai ta = AA; o. 5 24 = AACA de m ori. Aoeste m numero naturale ‘ot aven, prin imple Inn, una ain returile 0; 1;.<:m-— 1}. Dac ore fae dou ingest sunt dfriteatune!unul dino mimerele y= To Impure exnct la, le unde eoneiuei problemi. fn em only, Wor ext cone Solutt probleme alert form Principal cutie, dowd namere care dau acelsi rest. Diferen le, care ‘te dvb cu n, eave densemenen un namie format din elementcle natin craEp) 7%, Multimes 4, avand n elomenta, are 2° subrouliml, Daci x este un eae ‘ment al anitimi A, atuned vor considera malgimnen B= Af)" Multinon By avind m= 1 elamente, ae 2°~* subsallims care sunt totodatl gt subamulion te Ini A. Celeotte submit ale lai A sunt exact submmltimile at Bctora te adingimn elements. Aste vor impirt cele 2" submltin ale Ia A tn 2°" grupe disuncte ati {$;5Ufa)) (aeestea vor 6 eutile), unde ¥ eae 9 submulime a lui B. AlegAnd mai mle de jamitete din submlinile hi A (ocsten sat bee), conform Principia cuties, dou dinreelemente vr fi oon cutie, Dect una dite eleva nclas in celal Observafia 1: Arm ina cont e faptul eX toate submlinile lui A sunt: subline i Bs sobmuljile Ii B eizora le aug, ecteia in parte, elemental, berate & Dac am aloe doar 2°~*submulyini, nu ar exist print ee dow cae incase tna neealalts. Spreexemp putea Ina exe 2 4. Fie si p* dont puter care das selagi ret prin imptice ln 108 (act Incr cposbil aging suciente puter ale ai, speeexemph 108-1). Agadar, yh" — a2 = p(y" ~1) eso dlviaiil ca 10. Dar 9” este prim cu 10 seoth 10M" 1). Decl" so termi in OF mamta) serous Bd ol ink 1 ind ales Ua etapa, 1. Fie « oat mai mic mime dintre ole alse, Sedednd din fecaredintre folate m~1 numerealese pe x cbt m1 diferente diferite inte ea Aveste numereobinate sunt euprinso tre 1 gin 1. Dact vreuna date vale 1 cupeis8 in maltinen celoe me numer aleso alunel am obi coachin pune eX toate cele m1 diferente sunt di taltimen cla (dn care clude pe). Adar exle m1 eliferenfe vor putea lan m1 valor ‘tick [°$2) ~1 numer diet se gisoseprntre n~ [22] —1 numero, Pats hore ar tebui ew [2] ~ 1m [092] ~ 1 cora co este fas, droarooe 2243] > nm. (Vee! Jroblemal 30 3) Generaiare(Trorerna tid Sehr [lsei Schur, 1876-1941, matematician Herman). Fie & muir natura Impirjin yrul de mumore 1,2.3,.ek} be = ne ee A subalni, $i se arate cal pin sme dntre submalfin contine dowd ‘lemente a civor difrenté se off aceaghsubmalfine. Corot al Tree’ li Schr: Fie kN, n > 2, Impartio mule (1.2.3).50) iE suimaitimi. Si se ame of wa dinie acete eabmalfi contin doxd numer dferite gi uma lr ‘Obseruatie: Remarcim de asemenea urmatorad rete: Bie N= (2,.84,.0,2n}, n> 8. Denonatral cf N are exact n+5 sub S can elemente eu propritatea c& pentru aice i,j €8 evem i+ ¢ S. Rspansol la aoeasts problems este: Cale n + 5 submitiml de eeadinal As (nt1n$2..2n}; Age (ln) UA) ~ (Bn), pentea k-= Ti Aaya = ((n-— 1)UA)— 2s ‘Aas = ((e— I} UA) — Enea}, Anes = ((0— Ln} UA) ~ (ln ~ 1,2n}5 dass = ((o— Rn} UA) — (nb 1.3m — 2). Exist gio subinujime (ana singur8) de cardinal n-+ 1 care inept ‘onda din emus Tm (non Ln $2, .,2n) Nici subsume de Cardinal mal mare say al cu n +2 nu mai sti roprictatencerut{demonstraia eestor renaltate ve face prin inde cet 8 prin inde). Fie N= (2,3,4,.,2n}, m2 8. Orie sumbralime a sa care are ‘mai mare sou emul cx n-+2 confine J numeral inst suma a dowd di le este egal a trie. 76. Mai inti stabliaurmtoncea lem Lema: Oricare arf P un potnom cu cofcint iter = #8 y dou numere Intra cnrare, ever (ail) ~ PQ). Demonstrialemei Fie PCa) = aye" + @yaa"1 + 4 aye +a. Atul PEE) Pla) = ale” =I) + agate? + VN at aye —w) Cun (e—s)Ke" = 9) ‘cao m inte. mai mare saga rent colina le Din propeaitie avem: (d= a)[PC@) = Pla w ——E—EE———E (c= BLP) ~ Poo (¢=o)P@) — Peo Ayidarnumetee da, db, de ar tebui st Ge 41 9-1, aio dou dine crac egal. Conteedicte. 17. 8) Arti ok exist cl putin 3 dierent care sunt divine eu 3. Cele tice mumere dau prin impart la 3 unl dinre xe rei restart pues 0,1 or. Considertm prima cite mimes mumelr eared etl, «down hte malfimen numerelor care das restate cle mulimen numer fare dan fetal 2 Bvidentdiferenta dou umere aflae tn acca cut eae tivinbilé eu 3. Cele save numere vor fi repartizae ste Fre intro cite cel pain 4 rasere, Ne 3 it-una dint cui stot putin douk Jno ale ete, fe ete dou In ecae dine cle tri cat Intceare vate se constts ck exist rel pare de mmr ale ctor dferenje unt dive 8 3. 1) Been. c) Rime de aktat ch PITT: a5) Jn fancied pasta vom avea urmitoacele ean ‘Car I. Cal puja patra a aceeagt parte, deci vr fi gase diferente pare prin care dow milipl de patrw (sta deazeee ele patra mumere pot fe Masi Ma+2 dock sunt pare, e Mj +1 gh Ma +3 duc eunt impor). Can 2 "ret pae cate vor ave rel ciferente pare printre care una Ma, to Ipare care vor aves tei diferente pare prnte care una Tmorlare cas renulsh erat. 78. Subiin repeat wonton inser: [0 4): [fs Aol I arp le 1 pumere ere repens a mnerdr d from funda = TENET Conform Prine ete vor exit k ( k= TN 41), sii = TN astfel incht si avem: tts) (ou considerat b> 2), Prin urmare 1 Hy stk= De ~ (es) ~ tal) o Soluii probleme algebra robleme alge Notion = 15 = a — bg 80, Pentru feere 1 <4 <7, exist (—f; 5) asl act 2, = tn(). nyse interval (~F: 5) i ae interval eae, pute sigur exit ¢ 5) asl net 8,8, < fin weave Heal, ~ 691 . (A) waren te rfional etunct mumdral de frctt & care satnfec aceasta i pare ago +ay9 < Over 0 > ogy tage > aga don > aus taar >. > datas. wee in rata (1) relay + aon te pov gi deck toate cle 99 de sume +a, 4 = 1, 8, sunt pontive, ‘Solupit probleme algebris Acta te ai us eat pil Ca emp din urine 8 9,0 4, Fey mn one, Ark ul de deena {pty} ate mdr chon noes cts A. Siyten arty dpe tn Ao fotan do fy 52 yas ayy Bye y a) Prana aan he mia yy 5) eee ot xs alae coi y = 18 oe nf SyloniTy ean ‘Desai tf 4 tel i pa Sena an ed a ‘cna tae Diag os aoe ea care evident este irational. Contradictie. “ see Ger Fa se leone Sistem se ane he raion, Indicate: Se consider un mom irational eareare Atul este Aintzelementele malin fy, 2p, os ny, (20h Observapi: Bnuntulrimsne valabil pentru o iuiitate de mame a {inindwse coat de faptu ef rmlkimee numerelor rajonale este Jat con a numaerlor real ete nemumteabil, 86. ‘Toate umeree prime (ou excopia Bu & i evetualeaten cae ho, pri inp ak nl din eu 2s k= 1. ie 4 = Imultines numerelor prime care dan rst § pri impltie lak. Molin tnmerelor prime find infiniti, iar malfinile F find in nami Bnit contig unio muerte prin, eventual exorptn finitt Fie, sooth muitine, i sr < 22 < ‘numer prime, serise fn ordine stil erewciteare, Atel msnerlecutat oun ay Hay = EP, pentru n= 1 2, 7 “De ee et par gw MS, atu apres re posible, Pentru acest nena, cp, sexe 20 prose ‘Avatar da ese 20 ce prod ts ib umb factor pi Da toate clave 20 de prods contin nual factor pat dl wnt ‘ie oa 4 ——__————— - eee 4. Prin rare nisiun produs nu prin Spree la 40 restul 2 (nil eeelate posi resturl de forma My +2), Deck ext doui eare dau acoag est ‘Generalizatea pentru dn mumere, in le de 40, este evident Se pune problema decd, avid wn mumr oaecare de mumere, mei rinine enlabitefrmefin? 8. Seriom primele 99 de numere naturale nenule fn urmitoaren otdine: 5m 49,51, 48, 9, 1,99. Pentru a mu exsta dow elemente ino maltime fhve sm ib sata 99 sau 100, va trout wf mz Hm din slr dows mamere onscutive 3 deaseaenes, penta af 50 aelectate tcbuie ea din pine 49 tie peechi ete un mums ple nomial 99. Peatru a respecta acste condi ltclnies& algernsubmulgimea (80,51; 99}. Dac le Tuam po ele de pe pwtile pare 40, 1, . ni mai putea float $190, nstfel ens subsmalgme vind 49 de element 0. Fic A, eh Cos tei grup de ete 20 de persoane. Consider persona face are ete tual putine cimogting® nro singurs alté maltime. Pie perooann 2 tin gral A eae aze keunogtnfe in maltimes B. O persoank carecae y dine tele considerate af prietene cu x,y © B. Deoarece aren + 1 preteny, ane ‘in mlfimen C va avea n ~K-+1-eunostinge. Dar y are cet putin Fexnogtinge In. Alunel va existe printre ele o cuncsting& comuni, i ea =, pentru © 3 y Pareanele 2, yi = sunt ede cute 00, Patratelepefecte pot da pin impirtre la 19 zece restr. Conform Prine ‘pd cutie exist 19 umnee da cle 181 care da seta rst prin rape 1n'19, Acestea wor forma sumtimes lute. 91. Consider pie factonare ale clr mse de forma y= 0 +. Dhcs vena dite soste pls ctonare oo aft itrunul dine intervalelo [eek (onc fe mal propia mm eg de ume fepecth, in caz contrar in ealom ~ 1 intervalerimase: [55 523), —~» f sll cele mp eacionare le sumer Dot te eles wit in acl tral Fe > asl edt ft ~ a < pey- Dar acto muh do il ‘ta, De lt conn (02, Nameree prime mai mari dest te sunt de una din formele: Mig +1, ‘Mot Bsa +Tefia-+ My Agadar dow dia ealo tre mumere prime mal math 6 Selaplt probleme algebes hot tel vo 6 nw din multe A= (o2=My+l san = My-}1i} sou B= fx; Mp7), prin urmaze fe suma, fe diferent lor, a ultipty de 12, 98, Presupunen ei vistele lr, exprimate in numare integl, aut ay < ay $ ‘< ag. Presupunaan In continunee ck sma celor mai mari 20 de vaeste el mule 260; dec cel mai mle dire ele, aye este eel mult 13, Ca 1 <18 pentru feareéeuprins inte 1g 18. Dec ay han age $ ay +. +019 +260 $ 169+ 250 » 429. Contraco! 94. Fie mumerete 164,24, (P= Tea. Rostutle impistii osetor mimere In p sunt diferite, Acest Incr este ‘eoarece in eas contra exist ij asl inet ka = ja (med). Dei aff = far fivsbit eu, lucraimposbil deoaree peste prim ar @ 14 J sunt mil deep De aii, conform Principtuli cate, exist stl ict ka = 2 (med, Introg mai mie deci p- Acesta este moma clitatB, Unictatea: Fie e€ Z, 0 case este continnd, pri wrmare exist un mumir natural NY sll int © Seose $1€ pentru oveae = atl ici Innpiryim cerca in arco wale, Inoepinnl ew 0, marci pe cere punctol ‘ate coreapund puncteor 1,2, N+ 1. Aceate puncte sunt diferte dator faptulai eX este iational” Atunel dows dinte ele w vor alla pe un i ‘rl arce. Pienceste mumere atria gb Lian n= Jo Wy, Das On 90. Practc nis cere st artim eprint prim 100 000 00% trmen sin ‘edi el dvizibil ev 10-000. Ne propinem si demoustiim mat general: Oriare ar fi numa natural m, grinee primis m? termen at gral Fitonacet eel une dviibil ex re, Not en fs resto lnm (Fj, termenl de rang & al sil ornmim urmitoral sede pershi ale resturllor (Ps A Pas 5 Us fab os Bs Sosai Dot perecht (a5) 3 (6 d) sunt egal dacd a = eg bee d. Numi es Aistnct de estar obfiant prin impiizeala mente egal eum Aged pi brimelem?+1 peceeh considerate, dovk vor f egal. Fe (fs fra) rime ding earo se repet8, Ait ck acst prim eupla care se repel ce chiar pg «ln 9 adie (151). Voun demoasten prin reducere le aur. Presupuceas bral euphs cate se rope cto (fs fess) E> 1. Be (fe fers). > ke ‘unl u fai faea)- Conform propsititis de geneare a st i Fl vem urmftonrele: Pa = Fass — Fe, Phot = Fagi ~ Fi $ in egalitaten slot perehi considerate svem fui = fays fe = fy- Din sete og jobjinem fons = fenu deck aver ageliatatea de jexechi eas fi) = Ue fate contasic of pereches (fi fess) este prima care ae repel Deei (151) ste prima poreche din ica se epeth. Presypunem ck fs fa ‘ete prima perecho cate repeti pe (12). Aoeastaineamad ck Fy i es estat 1 prin impiie lam. Dock Fig ~ Fe = Feat ce divide eu me elem ei gisim on num terminat in 4 zerouri print primi 100000 000 de nal sri al Fibonacci. (Leanacdo Pisano Bigolo (1170-1250) canst ‘a Leonacdo din Pisa sau Leonardo Pisano eau Leonardo Boaacc! sa Laooards ionic sau simpla Fibonaci, matematicia italien, osiderat "cel ial ate alematicin cin ve al Eval Media) 4100. Vom demonstra urmitoralrealtt mai gene Gineraisnre Si $e arate ed dntre oriare n~ I numere fate pot aleye eof uct sua tora dvd prin na n> 2. Pentru incepa demonstrim uemtorl seta 6 _Solutit probleme algebret emi Diack afirmatia problemel este adevieatt pentru n = a gin = by ne este adeviatd penta n = a, Demonstratia lemei: A spune eX sume a n mumere inte este dive ‘0 mumarul for este echivalent cu a spune of media eritmetick a mumereloe repetive ent muti Inte, Fe date 2ab—1 numer introgi. Deoareceafrmatia ete adeviratk pentru mere cin 251 numereoarecre, ea va fl adeviraté si pentra 2ab—1 numero Mosrece 2ab— 1 > 261. Asadar din cle 2ab— 1 mamere pot alege b numero (ace au media aritmetek Intreaga. Apei dintre ale (2ab-1)—b umere rimase, sci (2ab— 1) ~ b> 201, pot slege alte & mumere care au media arumetich Inoeagh. Acest loca, adit extrageren act bnumere care au media aitineiek Ineag,o putem face de 2a 1 or davarece: Yab—1= (2a ~ 10+ (0-1). fan een mh 261 ap det # mee, fined wt 26 = 1 tnt. Dune tata bos ne ot pants = pa ox # me dso yt me cesses lh ‘ino a Gl pn Pon ue ob tne oes et Nn one ee nance Dae e donone sat pan ame pee 8 ofc, ey suing nna shan saetiad pe hee ae inn pre Bader pon ot tems ne erate ‘sar ens nko poet Carona toe ‘Teoma 3p mums eg Sle patel edt wma ro lp. Mir vo fone mone : Lend Fer nner ter bbb Re 91 101. Vom demonstra rezultatul genera: ‘ie © muljime A formatt din clemente. Dine submalinile acetela lye 2°! sublins eatfl Enc orca 3a abi wn clement com. Si 4¢ freed tots eee 2°? submadtimd av wn element ens. Solujie: Mulinca are 2® submultin, Din fale oriare tre submis ~ printe uml ale nu 4 lh mule vii, Soluit probleme agers intr submulfimile X 6X cate alent exact una (& eave complementara Iai X in report cu A): mulfimea vid nu se aft prot cele aloo; mulimes A este ales + dec printre submuljimil lee was X iY, tune XY a font dense. nen alas asta deoerece XY nue leas pentra cd mulinule X,Y 9) XP sar avea clement coma), ‘Mai ml prin inductio putem arita: Dac rut slese aubtauimile Xi, Xay + Xp atl si XXaN..AX se al rare cle seletate, de unde coneizia, Un rerultat aeainitor, din aces gen, este wraitorl Fie 0 mule X de cardinal n. Consierdm o fansite de submuim’ dis- ‘ince ale sale $= (Xy; Xa; Xe} ou propretatea ef oricare dowd on inert evil. Si ge avate ed memiral acestr submalini este nazi 2™-! ‘Soluje: Denim maultimea T= {X\%i = T;r). Toate acete malfimiale ‘ni T sunt diferite inte eles derive de elementele lui. Dec cara) ete eal nit jumtate din eardinaal mulgunl parlor hi X, Aged r-< 2°! ‘Acest maxim este ans. Fle A= X\(2}, 2 © X. Mulimen § care are 2°! ‘demente ex proprietatoa din enunt ete mune formati en lemeate de ipl RU {2}, unde 2 sunt exe 2°! submaulimi ale it, ute arta de asemenes: Fie F 0 Jamie de submalfimt cu elemente, le maitimié X. Interictia evicdrorr +1 main din Fete nevida. Ee arate oi intersect tutror alison fl Fete nei Solute: Fle F = (By; By; .; By), r+1 < a, din enunt. Deasemenea s < Cf IX), deoarece mutinea X'nu poste avea ma! multe sublime Presupunem ef a1, = 0. Fle Bt = (215 ‘vio maltime F € F,pentra fecare i= TF, ‘Anite By NF 0-01F Dick mu arf vidh coast multi, atune cele r+ 1 mlm ar aven cel tn un element comune aesta x, Atunel x, © By six, © Fy, contradict apt eb de ape ny 2). Deoareee 2, £ Mer ‘ete neat fF 102. Presupunem prin absurd ek fccare mum ete compas. Atuneifcare ‘us vaaves pain un factor prima mai mie deat V 209, nek ual inte tnumerele prime mai mic sau egal cu 43 care sunt in mums de 14. Day ave 16 nummere date oe el putin dou au un dvizor comun, contend 103. Fle = © R—@. Constaim un gir (saap care wh tla 2. a {2} terest paren irae, epi partes tamara i Sim 81s fe} 4, eupringe ine Og mate! ineat Her} - toy <3 toto) feo} + fal <2 a ooh fe ‘Acumn seat ct ete Aooarewe le-al< 4 entra orate nN, adie im se AAvedar @ este densi in mulyimes mumerelor reale R 4104. Demonstrisn prin inductio matamnatiek. Preeupunin cava k te M23, ae P34 y= —3, ‘sear doi prim intr e. Vom gs ayey prim eu once terion al iru Fle N = a,03..au, Conform Prineipinu cutie prnte mamerele 2,2). ‘eit do, Go acest 2° 3 2* cu a >b, care dan aca set prin impiive N. Alan Pda deer 2 ste divi eu 1. Cum este ipa reat oh N divide 2°" — 1, Atwoei fyyy = 2-3 ate prim eu deci ex Bocae dite nunerle aya,» 105. Fie 3; a; ty mumescl, prime inte ele oricare don, consierate i oan. Raioniin prin relucre la absurd. Pretupunem ct toate acesten sunt com> se. Fle 7 cel mal mie dvizor prim al Il aq. Deoacens a ete comps ca a nie dvizor prim al shu mao insusl. De ny < yay < 2n ~ 1. Namorle py, ‘are sunt fe impae fe numdrul 2, sunt in num de n- Dat evetn n 2 mime Inpare seu numiral 2 vem n~1 posbititi pentru mumerel p,. Contraditie, 100, Demonstratia 0 vom face prin induce, In cazul n= 5 ever 6 submulims, dintre toate evtmalinile de 9 elemento, ecare le grup in §peroch (2.3), 0.4.59), (C124) (2,8,59). (3,5) (2.8.49) (40, 3,4), (4,8) ((1.2.5), (8.4.5)) i conform Princip cutiet vom lus douk submlimi din acceasi pereche, ‘erste aviod un singur element comun. Si presupunem acum ei proprstates este adevie n26. Demonsteimn eX proprictatea este adevisatk pentru o multime A eu eee naa gin +1 submit ale ale, fn cole n-+1 submulfimi se af, in total, 3n-+ element. Conform Prin- pit cute exist 4 aubimulimt (Be ecesten A, 8, C gD) care at un element torn aM. oct nu arf adeviratt coucuale problemet auc submulinile B,C si D mai avea cSte un element comun,altsl decat a. Conforan Principia cutie Inclement bal ui A a continu de douk dintre oleate subimulin fe ele WC. Atmncis A= (4,5,0), B= (0,b,d), C= {0,5}. Dar st D are, conform ‘over Ia absurd pe care am fieut-o, douk dlemente comune cx fear dite wlanalimile A, B 3 C (teh elemento mee posits fe comune, in ce ea ‘le dou submalgia! ar Beale). Acesteelamente mu po deci $b nog, ores submkime earel confine pe af contin gi pe bs eewoe ic ¥ teuniunea eelor 2 submeltii ce engin pe agi 8. X aro 2-+2 eke hte ar coke n+ 1 = submits ease sunt incase in mimes MA fossa wvdnd m ~~ 1 elemente, Dar, conform ipotzze! dle induct, printre ‘lmulinile rimase exist dou care a exact un cleaent comin, ontendicte. ovieae ar 6 < ny a 107, Generatisre: Aritati cl oricare ar fn +1 numere éntret dt ‘cuprine inte 2+ 1 $52 +1, putem giei 8 prints ele care au suna nul Soluti geveralntit Vom demonstra noes renultat prin inductie. Bvident pentru n= 1 rezultatl este valabil deoaree singurele tel m pe cae le pot hia inte -1 $1 sunt, 0, I are an aut 0. Presupess pentru n > 1 acest retultat ee devi si demonste ‘i rimine sdevirat pentre n+ 1. Considerim, conform enon oli -X format din 2+ $ mumere fatrgi di interval Xe = [=2n 1 2n in Xy eal cinta, ‘Mine de analiza cazl in care 2x stn ~ 1 elements ele fa X sunt ‘vy ci rel sa patra intro oe al In Xaya\ Xo ‘Procodia ste Distingem dou ean 1) Print lamentele care nn ve afin Xy sunt 2n-+8 gt ~2.—1. Bement Tul Xs te imprint perecile emitoare” (1; 2n), (23m 1), wy (mee ‘ma oriciror douk numere din aceensi perecho opunindue hii —2n ~ 1 (0,~2n-~1), (=1;~2n) pin ln (nem 1) guna elementeloe ori puna lui ll 2n-+1. Cale 2n +2 elemente ale it X. difsite da 2nt vor fla io ele 2+ 1 pereci formate. Agar dous diate ele vor 8 in ere, dec! impround ex 2n-+1 sau —2n~ 1 vor avea sma 0 2) Dac unl dintre numerele 2n-+1 gi —2n— Inu go afl te X, fo 2n 1, atunei (~2n~=1;—2n;2n} C2. Alegem perechle (1:2n),_ (arnt 1) ‘cor sum se opie ii ~2n'—1 $i (0; 2m), (=I) —2n-F4), ms (Ces om ‘ctor sun se ope Ini Qn. Toate elamentele tui, cu excepiia ti 2n > 2-1 slewentual mente nu se Af nc "eaten cele ps (eutile"); ack 2n+ 1 ele Conform Principivlelexte concluia o enti 1 408. Remarcios ci funcia f duce un interval iotrun alt interval de ac "nite su in dow interval uw accel smd a hngimilc, Agedar J"(J) ‘interval sano renniune de istered Tungime egal eu hangin oh ist V © N aatiel inc (CJ) > 3. Atanci langimen total intereloe J, $y ny $%) este mai mace decét 1. Atuneiexsth-mumercle naturale k .K> patel nc (J) si f9(7) au cel pugin um panet conan. Dar fneia ‘te inoctivi deci oat = € J ate inet *-P(2) =, Consilerind n= ke ‘btin ons, 4 Solu probleme algpbe. 100, Consideritn ode 6 numere obnute dack ay ¢ (051). Bxstt print ele sok mamere diferite care dau celal vast prin impiryiv la 7. Diferena lr, tate este despa ats bonta tasty ‘sa, an a9, 1 toate mule, din alkimea {1;0;1), de andeconelsia. ‘Generaizare: Se dau mumerele naturale, fay ty Aric exit, ajvcn gs tote mile, din limes (1041) afl dust una me int sors retained racine OE eres tliaenn ty acre fe een Ainire aes interval conform Princpiu evi, dock exist cous, iryiye M. Langines uni astel de inereal este u- Dee: fe-y] < 10a <10™. Dar 2~ yest un mami do forma, at bvE-+eval tis aybe€ 2 ma toate mle atl ink fl fel < 10 111, Maiimes (a;--3a0} a> 0, are 2"—1submulimt derive, neve. Be tite cdl nr amid sen et Pon sie Ii ie aston Asi B- Maltimea cdutai este C= (A\B) U~(B1\A), ule win -(B'\A) se ineiege muljimen formati cx opusele elamentclor mulginl \A 112. Comsiderin urmitoorea progresiearitmetick BLE TBM os bm Conform Toneme fui Dirichlet exist infinite de moxere prime printeo ester, Fle pg douk dintte narton Atunel: p= Tbe sigs Lens, 3 Desi (n= 9). ” ‘Solugl probleme algebrit_ 0 altd soli: Consideriaa multnien numerelor naturale pastiQionatt inn progr aritmetioe, cere sve rain i prim trmen find O. 12s 8 (practic ete vorba de else de restiri mote). Mulkimen momerele rine ind itn i oate acest name prime alind-ae i cele nmi, el putin ‘un inte pari confine o infiaitate de mamere prime. Dou intr ele vor 8 mere p34 eluate 113. Presupunem eesti un mumie fst de mumere pei de forms 4n— 1 ie col mal mae dintre ee Consider N = 2?-3:5:...-p~1, unde produsul $-5-...-p eontine to ‘uumerde impare mai miei sau egae xp. mu paste & prt deoarove N > pede oparte, simu poate nveadvizcl pei mat tari dec paz nil rai hot, pede ale paves Imposti 114, Fie N un munis ttrog oarecae. Ari ok exist un aumit prim p> ute oct ps 1(mod 4). Considerdan uml natural m asf nest. m= (NPL ident m impar, > 1. Fie p cl mal mic factor pain am. Nic itso mmereo 2, 3, Na divide m, prin urmare p> NN. De ase (09? = —1(en0d9) rin dear la putes 52 obine: (wir = (-1)"* mods). Conform Tearemet fat Fermet,atematician fences, 160171665, even: GP = Hfmode). Din ultimele dou ea bine (-1)"¥ (dp). De niet result 1 = (-1)*9" adiok #4 este mami natural por. Atunci p ‘Ma 1. Dec am lst un mamas prim de forma M+ ma mare decit num natural oarecare 6 __ Soluti probleme algebet 115. Fie g € Nati inet ee mye. Dac exit p € N atl int ap > a5 bp > by stunel obtinem concusia. fn ‘a contra aver dy < dy su by < By, W BE g, PEN Decl by atu! rene coneluia dears gp, = ny Dees presupunsm e by, < by, Wie Ne Subir (y,) cate mlgint, agadae fonine un subsir constant (bm, Fi subyrul eq, Cancluaa reult observind eS dack Iam r# € N atl Inet ene > Gnes BVED bn, = Dy $F Oe = 118, Mflkimon A eso dons in R dacKoricare ar = © R exist un gr de Inmere (an)oox €-A asf incdt oy + 2, dack 2 + +0, Mai int aren exist un $i (agnor In A aif cit og + 0 dc n> toe, ‘Sum €8 0 < (a) <1 oricaro a € R. Deoaroee + ete rational urmitoarele nT anne dn interval [,1) fa; {20}; na} {n+ Na) orf iferive. Asaar exist dows prnte lo alate Snte-unal dtr iter ean) Dec exit fn) s(n) natural, (x) > t(n),eupriage vit pe) = femjoy) < 3 Nan: din = ola} ~ (Ua) = (le) ~ toni fm ~ Kina A Ade deduce ag +0 das > $0 Dae ind ek ag £0, obinem eX meinen Bm (hay: keZ,n EN) i dons in R. Cum BC A gh B denal in R,obfinem dens in R n 5. Solutii probleme geometrie 1. impstim ptcatal a patra strate deat en ator a dou segment eryediculare cstv pln central pStratal! care nese miloncel aro be in ptat (en fr). Figura $1: Problema 1 Conform Prinipilu cutie’ dou diate puncte eas nt-unul dint pate so latard idee e af lao distant egal col mult egal cu diagonala su de Inge Generaiare: fn intriol nat ptrat de atari se sou n? 41 puncte. Si sr ote nit dou nie le fee a dita de marin ° na fl de vat 2, Impiryind fcare laturn out objinm o imple a cub in 8 nb Intard 1. Conform Prinipilut cute exists dak Butur in acelagteubule, ect dstana dine ova fea mult gal cu diagonal, adios VA, ” he Fomor ie Generaizare: Bem 1 puncte 4 recta nro a de atari 1 ed exist cet pusin dona mivaeseiectee uncteasfel inci distonte dtr cle sfc malt, 4 Solan Pa on peel 0 cara a apnea Pal ea wo pu ec nse ees pte tS Qn pct dep ee, cola ade 0Q'= Vest ago Gian Sn ded (et Rg) 87 GAB se on gh * plant. Dack toate panctele de Figura 52: Probl 4 (a) i clone, problema « reavatk. Dac sunt de calor inte ele are noragicukare eu O sa Q, de unde Solan 2 4m figura alitraté toate segmenteletragate au lungs 1 ‘ont cle 7 puncte aslel int maven dou aoe i istanta 1 afl coorae, 0 Sol poems geomatse Pigura 6.5 Problem 4 (5) Consetim ci punctele By C, F rant dit eolorate (cu ete una din cole 3 ‘ubri avate la disponie). Punctle D, C, F sunt de asmence eiferit colrate, ac B31 D au areas culoae, Analog deuce of Agi D au aceas euloare, in uemare Agi Bunt capetele unui segment do lungime 1 colorate la 5, Vom aette ma int urmitoare: Len: Oricum am color punetele planus cu dow culor, wor exista 3 uncle eliniare ecidstante de acezateuloare. Demonstrafia lees: Presupancin eh planal eeolorat eu roses vere. Fie jnctlecolinare Q-M-P-P allate ls distant cgale, vel figura, asl ict M sunt vere. Dick uoul dite punctele Q, an 2 eats verde, ude NV ete hol segmental AEP, probleme esta inheiati. Ta eas conta, dec toate Inn agi problema cote ies Lachine penta ef, din coniderente goometrice tiple, O-N-R sunt egal depirtate. © ie A-B-C tel puneteeainiare sj echidstonte de asengeuloare; fe aceasta tuloarea rose, Constriim trunghialecllateral AACF gi fe B, D sie B mk lose lature, D milcal hui AF, (vel zw), CA Figura $4: Problema 5 * Dacis una dntse punetle D, B sa F ese vyu, problema este cheat Spee poe sours uuu dine tnghinile eehlaterale AABD, ABCE sau ADF {remeron “cw cntrac ADEE va ve iri de aeeg ela, anu Pent paren dows problem vom proceda at Inti plan in bent verte de lime 2 le clo tera cx unite ele dou clr vite la dsp (eapa in sng ere Into bad vo color chase clone caa beni). Com ep Inge nh us og eral de ata ete&e t ecnera de ltr reve dbo wc, dec te de ners cane in schinb tem demonstra unin rena Si se ard ooo a nor plana eu dou el eta chided ter {a0 V8 cay art rr de cna are Demonsnsje: Fe puncte Ag B, [AB=2, colerate diet (A dec rove B verde). Fie un tomb ABCD de atara3a cdr dagoalt AC ete Inge 2: Te de aseespunctl asl cst ADEB ect ‘sw va sven lungimea V3). Putem presupune ci punctul C este ragu (daca ‘hr se fen stint slr cnet einai sole ps ua) * Dar ural te panel D stn ete rp aun ein un nonormatic d ltk , ABC aan A ACD. * jn cax contrat ambele sunt veri gl objinem triunghiul monocrom DEB de iaturk V3, 6. Considerim un core mare al fre eare trove prin dou intr cle 5 pun inte celelalte 3 puncte, dou vor 8 Int-unadinte emsferele determinate ‘rou respectiv. Desi vor B pati in sceeal semisferd inchs 7. Cu ajstocul a 3 cereti mati ase in tri plane perpendicular, tmp sera 8 supradee egal. Douk dintre ele a at peo tel dosuprafat ‘uli, prin urtaee, lao distant de maxim w, 1. Vesi Problema 16 de tx algebri Aces cameluse mal rane valabils in cama in cae considerin S puncte. tn shim putem stabil uaitral relat: ‘ie 5 puncte sn plan stl inet orivare $ sunt necoliniare. Feared ” Sol uncle se unese fine ale fie print line roy, fe pinto lini alba aatfel Init wie wm ripe 38m formeze un trivnghi monocolar. Si we erte of 1) Din feere punt pleacé exact dou lin roi dou last; 1) Segmentele rit represineé 0 liniepoligonalétnckisk care confine Lote cele eine puncte i ecestdproprictate este wall gi pentru sagmentele ells 9, Fini in plan um punet A. Pri ast punt dace gave parallel ee ase shop considerate, Ver bine afl gas drepe dete cre sunt concuente Jn A, frm ete 12 gh Sn jurul punctatus A. Cum sun lor este 300 ‘ovetistn unl x mara main de 30°. Generalizarea este evident. 10, Tre drepte pot determine un mumar maxim de regni in plan dack care dow sus parlale gt oricare tei ma sue concarente, In ast en ele font dreplele suport ale uni tiumgh,imprindl asf! plan in eid (vee Agu) Figur 55: Problema 10 Dect pater & sig ck ex douk puncte tn acct rgiune. Se paate atta ek numnal maxim de rennin care poate fips un plane ajatoral a dzepte este SE 41. Asadar: Cenenzare: Fe nérepte core impart plans i mai mabe repen, s0 ane eer om tris +2 pte fx get plan, ical eft pe tina dite ipl, ent dud pent flat reac ean Consider un punet A dinte cle 200, Pe seeta gi pe erlelate 359 eare ot coc in 80 re de ete wn gral color cu rogu. Cee 200 puncte fonsiderate se alk prnte ele 860 puncte rogi. Punta rgi Ie impirgim in Ii geupe ce eite doua punete opus. Conform Principal eset exist dou treet 200 ponete alate bn tea gruph dei sunt dieteal epuee. (Paton eee. sigur ehiar de exstengaw 10 pereci de puncte diameral ope.) 12, Fier numira!ltuslor poigomll convex. Suna unghiuilor sale ext lon eta 180°n = 180"(n = 2) = 260" Print unghie exteroate vor exis cel mule 3 btise, det cel mal sir anctiteinteronce. 13. le Af an punctoacecare in interioal petrulaterulu convex ABCD. ulin ual dine unghiuele CAMB, ZBMC, CMD, ZDMA ave oa yu ‘com m{ZAMB) 2 90". Atine: punctl Af ete scopes de sel Alametes AB. 14. Duce te lin vertical la stan ogle, care npart pital in Areptunghivel de igime J gk langime 1. Ducem o ie orizntalk tute ptratal i opt dreptunghins (vezi gua), Recare avind asin Figura 6: Problema 14 {in interior uni astfel de dreptunghi exists patru puncte, determina fet wn patrulster de rie el mle. 15, nsw ovcae dou pte ale table puten gal un gir de ost 16 vein (inelsiy colo dous)- Agudar itr patrifelut cu mumiul 1 3 cel hil 64 exist 15 diferenge ale numerelarfaerse in pte vein. Di Allene maxim ar 4 atunch ar trebut em dferenta mumerlor 1 gi 6s ‘masien 419 = 0; desi ennchiia. 16. Lema, Duce prin central unui pitrat dou crepte perendi lnnpdrind ste tral in 4 fig. Atanet a“ ——eE 4) Dac dou puncte se ft fntr-ame din repianie format, lnginen sep: ‘meni dmtre cele dowd puncte este cel mall eye ew ltr patratale 1) Daci tet puncte se aft fn tei rep ert dintre cele 4 an interior pital, trunghia format confine central pltrtuti ‘Acum trom la rezovarea probleme. Ducem prin © (centrul ptatull) © rarall laces maf apropint Inturk a tringhiul considera. Fie aceasta dy. ‘pei, pein O, duoem o peependiculars pe dy. Fie accasta dy Suntem ase in cone lemei. Cle tre veri se vor ala in maxim dak dint cae 4 Imgimi, Conform Prinepilut eater dou date ele vor fin crea ree, pin tumare datanje diate ee ste mason 1 11. mpi cul de Isr 6 in 12% = 1728 enh de aan. Ni un Crna eu de nt, deocos rb nls net d tr, tu poate ave cern intorl unl ea de attr fcr fi pe fot, (Ca are ah ef en Av n total 12 2.6 (12 8) = aR turd ats | pe tek. ada cle 100 de cur mate 6 pot te cle 101 cn le curr de atari 1 in neil el de latrk Ti douk dsr etre oo at acl cub de nur prin rar, fr cetrl nated s pe una din ee ea 18, Vom avita c& m puncte interioare determin 2n+2trionghiuri care an Inusoarele dijunete 5 care acoper in ntregime ptratul, Veriicarea 0 vom for prin indie. Penta earl m = I vnriScarea este evident (le 4 vrbari 4 uml interior determin 4 triunghiuri dee 2-1 +2). Pentrn pasil 2 dock DligSen wm part, asta oe va alla fa interior woul tunghi deja existent ‘mis numieul de xing eu 2 Prin uma, cle 1999 punete interioare i vrfurile pitratull wor forma 21099:-2 = 400 tianghinn disiunte care acoperi peat. Dar aria prt ‘ot 400 det eit trimghi dean pti 19. Pe lingA stele eolorste marci si simetrcale in rapt ca central ale furor colorate. In acest moment am "inwemnat” mai putin deeds jumitate in coe. Agadar exist un pane: sf simetsicul su ff de centru eace mu sunt orate, det exit un ciamotry cage are capetcle neolorale 0. fmpietien peal fn 25 peated lau. Eesti nul dint ae care inkine 3 pete, un ate de plea find acopecit de eereul sw ercumserle ‘veil rat este cbs. Dat ply < 4. Dec coreul caro re central So eatrul Eolugil protien geometry _ pitrablul eae confine tri puncte gi de rst $ eae ea eat. 21. Impsgi cub in 1000 enbart de Inc $- Conform Principiats feet tei puncte in colagt cub, aot cub avid rasa sfrel creumscrse jh Tel ch exit for de ras 4 cre contine 3 puncte. 122, Prine ox 0 shi atrale ale piramidel ost 5 de accel culate, coast culoareaalb. El an cite un capt un vf al bagel. Cele 6 vii ‘hast detrei in otal 10 segmenter pine acestea eel mult 4 sux lburh pligontl, deci eal pata 6 sunt dagonale. Dack piste accstoa ete und "Mone vent rang de cuore aD fn eax contar aver determina de O diagonal care woes ele § vier alo bagel un trunghi cu Intute alba 28, Consderian un cere C* eonentse eu Cg de xaxk 16+3=10. fn acest [um consider ele 650 coroane eleulae care an drept canta cle 650 pu tle po dseu! C,congruente eu coreana cixculars A, Sind sigur cunt inclu thea! '. O croaa crelars ate suprafaa Ox — te = Sn. Cele 650 de os fut eupraijatotalk Sy 650 ~ 2250s, Acnste coreane sent in Fntrepine cul ete ao aupeatata 192-1 = 361x. Conform Principal eat , ( Propritate 2), existh un ponct-X al i C’eaze este wonperit de 10 ear clrelar. Fie Ot, O2y~ Or cle 10 cute le coroaneor care acoper panctl X ( ‘role se nS pe deal C). Desarece coroans de eautra O, acoperé ponctul tune’ coroona, egal cu A, de entra X, acoper’ punetal Os, Dec “do castra X, egal eu A, scopert 10 puncte. (Bvident mu 650 este mini ddaggeeu do git all). 24, npn cerca rigononnetsi In 24 ance egal, care avid 18 ineep ‘ punetal 0 al carat (punetul de pe axa Oz al ercul}). Marcim pe cere punctele Ay day» Aas ast inet Ingles ardor A, ‘toi, une Aste petal de pe cere generac erespunzind hui 0. line acesto pane se vor ala pe aclaglaze al exclu, de masa 15". fea Ay An Doc oxi mn € NY, 1 Sm < me S 25, ast etm ~ n este mal dct Inna aeeul de 15 ‘Aguas to(m —n) igh? = shy Dar 1 Samm 24 deci mn a ote maria ett. 26, Inplitim peal in 2 pra, care avin aria MQPemn?. Dac i 8 SEES 1-41 paseo, tel dite ele vor fin aca pitrie,del vor f vase unl Ivungh do ari col mult jumétate din ara patrjellu, died A=". Din Iooteat AS 0 dec 5000 ors lic? > 280, Desi 7 minim este 50 26, Consider fafapolidrull cre ae cel mai mare muni de atu. le woot mami. Ce fete aliturateaceste ete au un nui dela cup Inve 39m. Conform Princip cui exist dou foe care at acelag mE delta, 17. Consderimm n numiral de ealoi. Pieato cee are punetele eoarat fn ‘ol puja oculoare gf cel mut m, det exists 2° — 1 posblitig de eulorl ponte nce ceeufu. Fle 2" cercust de raze mat mil docst V2 gi de cents O; Printre acstea exist douk care a scolous culo For yl Ry < Fy rale vie douk ceeur. ‘Thebule sk gsi un punt Y pe cared de rash 7 cae ah sinc reli par). Putem 6 sigur de exstnta acest punct deoaroce ecuiaanterioarh i ne nnseata a(Y) are solute: a(¥) = Br—s € 052%), 9%. otesuient st demonstrim c orice pontagon convex cu vifurle puncte Invik, conte el putin un punet lati! in interior. Pusem gis un pentagon ve al ll un punct lticial pe atrium pertagon ma ne (e fs) Conforas ProMlemei 12 existi un punet lticlal care este moe al una Inyrvnt care are eapetele virfur ale pentagonnlni. Acest punet este ineror tants denne pentagonal ¢ convex! n plus:mu poate pe atu pentru inn exit punete lai pe atu ‘Mune <4 | Aci ume minim este 7 " SS promiame geometsie Dac n = 7 atunei dint cele 49 de pitreleexlorate cu cle 3 euler vor ‘exit cl puyin 17 care au scweagiculoare. Dintre eee 17 peel coloete la {el vor exist 3 pe acweasi Hines respectiv pe aceeagiclaan, Dac n = 6 sltmagia sue adevirts. Exerples _Solutit probe geome 9 e@oregr Ga att mat mult afrmatia nu e adeveats pentru m <5. Acet Incr ve poate voce in subtabele ale acttui tabel 80, | Considertm o rete de pitsat in sect plan sun deeptunghi deforma {4X82 de puncte din aest plan. Flecae coloand ete format din paton puacte colrate cu 3 culo. Prin rma exist 3* = 81 posit do colony petea fccae coloand. Avind 82 de coloane, conform Principal ewes vor ante ‘tou colorate identi, fccareavnd ete dow puncte, alae sescy lis de woeagculare, deci care frmcez’ un dreptungh cu vfuie de aces cloate, Asedar putem deduce urmitorl rena Panctele wnetrejete de pdtate sunt colorete cu dou cular, $i se erate i exit un dreptunghi cx vfurile fa nadurile rele! ealote cu 0 sind lear. esultatul urmitor se deduce isdiat dia sees: O rej init eat color cu dow elor. i se arate ct exstd dou pte orisntae i dou dreptevertiale ale edror inkeseri nt { puncte seca exlore SA. Colon sacle smetrioe faf8 de centr ele arcelorccorate. fn acest ‘woment nu tot cerenl est cloret, Mai mul, orice pnet calor nce puncea \taaetral opus colrat si rice punet necolorat are panceldiametval cpus ne ‘ora, Cum am convent ef exist puncte meclerate, remit conclure, 32, Phas cercul cu centr i origins axelor de coordonate, Dac punctul ‘le coordonnte (a) este colrat,atunc colori vate punctele de coordonate (a:b) Anti! suntem sigui el existk ines puncte necoerate, Pe os ect le punet (a; y). Atuneitonte puncte de eoordonate (419) sunt neclorete ‘sunt vasfarile una droptugh igri in oe Solutii profleme goometrie 488, Plas sera on central in originen sselor de enordonnte. Dack pune ‘de eaontonate (a:b) este colorat, atu’ colorim toate punetle de cordon nate (Lo,+h-42). Anil suntem sgiriok exist ine puncte clara (vera ‘oleae doar 96% din suprafata afer). Fle un astfel de punet necolrat de Sordonate (xy: 3). Abie! toate punetele de coordanate (x; ty;2) sunt tnsolorate sunt vifarle unit poralelipiped deeptunghie Ise in fra. 34. Fie A multimea punctdor arcelor care sunt eolorate. Fie 2 mui Deter afate pe os, diameteal opuse punctetor din mutimes A, Fie Inuines putetar de pe eee obtinite prin rotirea eu 0", fm sensl econ ‘rasorni, ale pantelor muljinil A (scestescorespund punctelor obtinate rote ct fn eel aves arcalor de ceasornie no punctlor mala 8) ‘Anilog, fe D nultinea punetlr de pe ere obtinute prin rotires cu 0", sensu invers acco de ceasorne, ale puncelor mulimis A (acest Dnetelorcbinte prin rotires cu 90" i sna areler do xasorie le pu ‘ulin 2). (Colorim toste punctalo multinil AU BU CU D. in acest moment ma puncte cerca! saat eolorate, Fie X un punt necoloret. Atul acolorat fisi panera ¥, dlameteal opus, punctle Z gi care sant la BO" pe ox Dunctal X. Atuach XY ZT este pltratal clutat Gneratiare: Unale res le unui core sunt clorate, Suma iaginslor ‘wor aren colorate este mai mied decdt + din fngimes cared. Arata fenistd wn poligon rgult cu n laters nacre & care cate ore toute eu coor Indialie: Color are, eoregpndtoare ctor deja clorate, obinute oie coved ee BE SE SRL 85, Ver exeaiil anterior. 30, Consens prielele de ltuntmprenni cu punctae ale cro diets in In pirat sue mal mil dat § (eat igure). Avia nel ste de suprafete este $= 1-44-44. Asa total a aor to fgus (piteatleimpreank cu extensle lon) este cal mut S, = 360+} 30x 00} 03,2 = 456 < S00. Agedar exist cal patin un panct in inte “deplunghintat care este coniral unui cere de rat caren are puncte cm ‘x ptratcle considerate ____ Solu probleme goometro O Figure 67: Probleme 36 37. mplirtim dreptanghol fa 5 pica tn fgura asocat. Figura 5.8: Problems 37 Conform Princ cut exn douk pete afte nec en, de ‘care so afld la o distant de maxim v5 unu! fati de celilalt. 38, Pe care ata atultr corn n deptanghi spe nein stated ca dimes lar eapetivk wf Suma arr er ‘tepungit connie ete PAE = 5. Cum ele patra epg a pce comune, condone eit un pune ier paar oe ‘opt cde nil um retunci Un nti dept ete entrar era ‘inn ieee tn ptrltar Wie O centr mes $10, Oa Op centrele conor. Considecim dow ‘omental alii care limba mitarlul ae alin poz diametal opus. fom arta ck intel inte aceste momente are le eringa- le pore vrfurllor minitareorint-tnl dintre momenta, Ay, Aa, Ay 1 By, Bay By la ella moment. Se formeasi triunghiule OAL, care a ‘x modiane pe 00%, Au le inaltiile : A, -+ OB, > 200 (pat fi eoliniae 0,04, Au, Bude aten nv oe negate), ¢ = Tym. Prin adumare membea Saupe proreme goometie 1 mesnbr objine Ssoas Sons $00, Inyalitaten find strict doarece patem presapune cata ale prim mome stl inet eal putin puretele O, Ay $1 nu sunt eoinae, Alun cel putin una dintre sume este mai mate dest suma distantsor Ja centr mese a centeleceasrir. 40, Ble, #€, co mae a dnc une clencllr Ske! Bi ‘pen ce Daca atin om one oa mal mi sund de pe Sasi, utuorsemetlr tablets el puta w= ade eon *"Proupana ols < 9 Sto ee sn age pl lo) Aan etna peta ram epee s Prot cre test ssa een ea patna dere: cre stv aBhp ia cu uma dnt alc abel are ropraen aroun cee i po cate na el pul Suna etal de pe onl ae sonia pe el pain #6 susie pe ll na cba) Sun telnet, cid sana slr ep col pu me +oee 52 (no +e 41. Cole dow discuri so pot supeapane in 200 de sodas (ricatativ vee figs), iecre sector al disculul mle coincide la euloare in 100 din cele 200 do ‘mode psiile de suprapunere eu setoarelediscului mare, Dei Tn total sun 100.200 = 20000 de perehi de setoare afl coloate noe 200 de suprapunel Donibile. Agar Ia ol pu{in una dint suprapunes vera minim 288! — 100 ‘de wectoae care au ace culoare cu cele crespondnt pe diac ae. ” il probleme goometrie Figura 5.9: Probleme 4 42. Flepolgunl Ay Aa. Ago, Presupunem ok na exist doo cfr lb care st incadreze pe un ltl. Find 25 abe qi 25 nore vom aven donk vf con. ect lbe, Fie scesten A i As. Consider snmitorie lini de vf (AusAsiAwk (Aria Asay (Asi Ar): (Ania wos (Aas Auth Ce Anh asia} i (Ate). in aceste 24 de tulim se af 5 do vactur albe, lect dou vi abe 9 giseefn cee male, agadar contradic 43. Observiin eX mumiral total de sectoare eolorata ex negro, dn cole 100, ‘presints 60%. Veem ak aritimn cf exist o secvent secatie care conyine exact 24 de setaure neg. Cele 24 de sactoarereprecin 05 din cele 40. Aceasta m eo simpla coineideng de procente Presupunem ci nu ar exista 0 secventl de 40 de sectoace cares conn act 4 do seconte neers Vor aria ei, datoia eesti Yeincidenta” de procente, eet scvente de 40 de sectoare care a mai pusn de.24 de secure were salle care at ma nul de 2h Intr-adevr, evem sn total 100 de seovenge difeite de cite 40 de sectare. anv sector negru pase & 40 dite secvente, der ele 8 de semente neg (21 2400 de apart. Doi ave o meted 2 de eparfs de ecioare gre pen Focareseowenti posi de edte 40. Prin urmare, cum nu exists seevenfa care i contin exact 24, om probatafrmaia, Acast ulti afirmaie est echivaleat ca ware fap Dac aver mai multe wun vale afl inet nu exist nick wnul eaves {fc egal ex mca or artic ate eed el pain aa cre este mai mi ect media lr gi cel pufin unl are este mai mare det media lor, ‘Agar exist dou secvene de cle 40 de aoctone, fe aoestenasoentele 9s ‘Solutii probleme geometric ai, astfel inet una contne mai mult de 24 5 cena ma putin de 24 toave ngre. Retin 5 pa en pas. La fecare pa asin tn stor ip ltl pentru Sune fe mein eI, ma scimbam nil, sas sem eu rumiulde sectoare neg ale seevenel. Facem acute ro pind se suprap scovenja 8 poste 7. Ara pornit ew S avind mai mult de 24 do sectoare, fm ajo ein vig le mira de aetoare neg do cel mule unl Secven{ care confine mal putin de 24 de sectoare nege. Prin urmare, la ‘mooient dat a exstt osecvenié cu exact 24 sectoare negro. 44, Deseniim un ere concenteie de aa 1 (det care ae diaeteul 2). Ds rao in corel mae, mptind ate egiunon caprias inte ewe mare gh ‘mien part eae. Agudar ar impistt cereal n 9 prt feonform Princip rule exist dogs panete situate in sceagh reine (ea Burs). Figura $30: Problema 44 ac cele dow puncte se afi in cerel de rank 2 atane! distant dinte ons puncte este cl mult 25 problema ese nebo, ‘Dact cle se aft in una dint eslellte § roguni ogle intr el, stu listan{a maxim dine cle douk puncte este Iugimea segmentuiai AB. ‘nesta flim din teiunghiel AOAB, aplicind Teorema cosinusd, sind m(ZA0B) 45. Hie ry; ra: ry rao ccurlor din interior pitatui. Deoarece lunghlor creator ee 7, doduoem e8 sama diameter lor ete Dp bra bat o _Sebuif problenss nots} Pe usa dintre lature pitratulu proietimn toate cercutle. Proietin Hct ere pe laturapltratlu este un segment de lingime dametal cereal rspte= tiv. Agadarpreeclecereurlr pe latura pitti sat n semen de san 8 Ignilor 2. Cum lature are fungimea 1 atune!$ inte sgmente at un pune ‘omun. Perpendiclara dust pe Itura pitratuei fn cel punet ve intesecta Generaiare: In iterioral unui pitrat de later fs ofl mos multe creurh ‘ave ox sua hnginilr L, $a se arate 6 exstéo drag cure ot ntersectere cel putin [2] +1 dinte corte cera. 46, Fioo dreaptia perpendiclark ped. Proectim pe fecae inte ole doa repre toate cele 20 de sogmente. Fie, reepectiv ys, lunge proetilor cela heal ea segment, ¢ pe dreptele d,reapectiv a. Atuoci 25-4 th 2 (oe formeazh un tlungh! dreprunghie (eventual degenera) car are Iunginilo Inturlor xi 1). Prin adunare membru cu menbra se oboe Ge tent tem) + nt int + me) 220 unde reztt eX una din ume, conform Prinspilo cui, este oe putin 10 Fie By teat nan 220 Dar toate sgmentele xs, prostate pe dreapte d, sunt indus intrun segment ‘delunge 10 care repreants proecta cereal pe acca dreapti. Deel vor exist ‘lous preci eare au un punet comin. Perpendicilara dusk pe d prin acest et comus intersetanzi dou dintre segmenele date Gneraiare: In iteroral ered de rast n se ufti4n de sgmcnte de lungime 1. Sie arte ed se poate duce 0 dreapl parses son perpndicalri 0 droop doth, care intersectasd cel pufin dowd seqmente, 417. Mal ini demonstrhon uemitoares: Temi: rind date m > 3 puncte sn plan, oricare tet necoliniare, nu se ot forma teat malt de n(n 1) tigi’ isosooe (ielucénd elaterae) ‘uj printre acste puncte ‘Demonsrajc. Pe meliatoarea unui segment determinat de dou din este Inmete se pot aa eat mult alte doak puncte (in eat contrar ar fi 3 puncte Calinare). Agadar vor exata ol ult dowd trunghiue iswcele avin ca box woul din gabon; deck vor fel mall 2C2 = nfm ~ 1) tung rose eu isla rinse cea punete. %6 vem C= MEIK0°2 tstunghiart, wom aves cel ma: nin= Yn 2) 6 nin 1y = == 8 leiuaghin’ seafene, ‘Avind 80 de pune clorate ex 4 eulor, wor exist 13 eae sit coors scons culoare ‘Asdar considerind fn Tens n & 15 reoull existent a el pati 190 unghir sealne, 48 Me Avda dog un dodecagon regust iseis Ta ce. Thunghiun AvAvdo, Aadedioy AsdrAur $i AudaAva sunt ecilaterale, del 8 vastus (ale 12 Sant colorate fe galben (ve! Sun). igua 5.1: Problema 48 CConsderim pitraiele AyAcArAio, AadsAsAus i Axdeo A Conforaa Pry eter el ate pate ae ol pi ek ar one oe 49. Vom demonstra urmtoarea generale Fic dons potgonte Ay AawoAy $1 BBB, unde m gin sus urate impare. Ducom toate segmentele de forma AyBy, <= T sir ae formese tick wa triight monocromatic. $4se arate cat cele m+ Imuchi le pentagoandor swat colorade cx acer caoor, ‘Solan ener: Presupunem prin reducare la absurd eS aturile poligonui A a.-Ay sunt Inf eolorate, Aan exist dou mucitweene, Adz 9h Aad, dent ‘olta,rgie, epoctiv verde, Presupue fra pirdegenealtaen ch Dy Aa ote opie Deoarece a spar triunghiuri monocromatie, rule By este a ‘Aritimn of egmentul ByAy este verde. Dac ar 6 ru, atunei ByAy ar 4 verde, prin trmate uuul dint trunghiurile A,B sav AB,B,Ay este monccromatie. ‘Agudar am sitar dal segmencl By gest roy ane! Byy este vende ‘Analog, dack segmental By Ay eata verde atuncl Bay est ru. Dest Bg sagt» Body verde ~> Bap rogu +. Bye rosu -> Bia verde (am nut cont em ipa). Conraditi, agedar nu pot ft dou lata ‘deena diferite ale aclsiag pigs. imine sé arit ck acct dou poligoane su mucile nlrate ct aceash ‘uloare(caloarn de I primal poligon eaeecai cu cra de nal dole). 150, Cousens free cn centrele in aceste puncte sd ra J. Toate acete sfore sunt inehse in eubul de Inturé 10 care are cclghcenisw eval cubulul Consider in poten, cele douk cubur evand fefle parade. Dac cum volumal ‘eununi calor 1960 de sere nu depigeyt voluml cubulu de atur 1, obtinem: 1960.45.32 10? ~ 000 12 5.08. Agadar sferle ma sunt disjunct, Dac& dows sfore se intarsocteasi atuocl centale lor, care ent douk diate punctole conslerat, su Ln citant mai nik dest 1. St, Pritreovcae trl panete avem dou eolrate la fl, Not eu Ay si Ay fede doud puncte colorate I fe, dc sunt chiar to eolorate In fo him oak ‘itr lela ntmpare, din triplta (Asn. Aaya, Ansa), Consider cele terechi de sogmente monocolore. Ficeare distr acete n segmente este paralel ‘us dine cele te Iatur aletranghiulu initial. Atone exist tn aceast ‘nuline ool pin [3] +1 > 3 Seqmente paralle (am considera Itura tre ‘inghulu ual pe Tang olen segment. Dar acest segmente paralee sunt ‘arae fccare cu una dint cle dou elon vate la digpoaitie. Dec al pain [il] oe sagan pina cro sept tee i ti cope n doi dine acoso vg or Sa ae 52, Procedian ase lori cea mal mek dine ature fcr trang ein ef exist un trun eu toate Ital colorate (dex cea mal ‘ware Ist a acest’ tranghi cos mai tle in alta deoaree este eoorat), Fie ose 5 weprente care su wl dint eapete pe A. Pinte acaten exh li care sunt fe eoorate, Be neclorate 1) Duck AB, AC, AD sunt clorate cum una dintre lature ABCD este corn va renal cviaga. Duct AB, AC, AD mu sunt colorae, atunel BC, BD, CD sant eolorate ‘tours rianghiuile ABC, AABD, AACD trebui a sib una dtr ata ‘obese. Decitriunghiul ABCD axe toate lature colerate, Problem este asemndtoare ea tip de judeen eu Provera 11 Cuprins 1 Introducere 2 Principiul cutlet tn Algebra 2.1 Probleme rzolvste 22 Probleme propuse 3 Principiul cutiel in Geometrio 5:1 Probleme rezolate 52 Probleme propuse 4 Solutit probleme algebra [5 Solutit probleme geometrie 4 2 a 6 a

S-ar putea să vă placă și